Thread Rating:
  • 0 Vote(s) - 0 Average
  • 1
  • 2
  • 3
  • 4
  • 5
usmle demo mcq - usmlenew1
#11
<1>A 72-year-old male is noted as having a 9-pound weight loss over the past
few weeks. His past medical history is significant for oat cell carcinoma of
the lung, without known metastases, for which he is currently undergoing
treatment. The patient states that even though his wife is preparing his
favorite meals, he is not hungry. Which of the following would be the
best treatment option to improve his eating habits?
A. Amitriptyline
B. Megestrol acetate
C. Methotrexate
D. Neostigmine
E. Prochlorperazine

Answer
--------------------------------------------------------------------------------

<2>A 45-year-old homeless man has a chronic cough, a cavitary lesion of the
lung, and is sputum positive for acid-fast bacilli. Which of the following
is the principle form of defense by which the patient's body fights this
infection?
A. Antibody-mediated phagocytosis
B. Cell-mediated immunity
C. IgA-mediated hypersensitivity
D. IgE-mediated hypersensitivity
E. Neutrophil ingestion of bacteria


Answer
--------------------------------------------------------------------------------

<3>A researcher is examining the distribution of an ion channel protein in
the kidney. She incubates slices of kidney tissue in a dilute solution of
a specific antibody directed against the protein, then uses the
immunoperoxidase method to localize the ion channel proteins.
She notes the presence of brown pigment in a population of epithelial
cells, which on closer examination, have a brush border. The researcher
concludes that the protein is probably present in cells of the
A. collecting duct
B. deep portion of loop of Henle
C. distal convoluted tubule
D. glomerulus
E. proximal convoluted tubule



Answer
--------------------------------------------------------------------------------

<4>A 38-year-old pregnant woman with a past medical history significant for
chronic hypertension presents with a blood pressure of 158/105 mm Hg. Which
of the following antihypertensive agents would be most suitable for initial therapy in this patient?
A. Bumetanide
B. Fosinopril
C. Hydrochlorothiazide
D. Methyldopa
E. Valsartan



Answer
--------------------------------------------------------------------------------

<5>A 3 year-old boy is brought to a physician because the mother notices tha
t the child is engaging in less active play and tires easily. During physical
examination, the pediatrician notices that the child's thighs are larger
than normal for age and that the child cannot stand up without using his
arms to help. Further studies demonstrate a defective dystrophin gene in
the boy. Which of the following people in the child's family is most
likely to also have this disease?
A. Father
B. Father's brother
C. Mother
D. Mother's brother
E. Sister


Answer
--------------------------------------------------------------------------------

<6> The leukocyte pictured above stains intensely with acidic dyes such
as eosin. Which of the following substances is contained in the crystalline
core of the granule at the arrow?
A. Lactoferrin
B. Major basic protein
C. Myeloperoxidase
D. Histamine
E. Tartrate-resistant acid phosphatase


Answer
--------------------------------------------------------------------------------

<7>A 7-year-old boy is brought to a physician because of a nearly confluent,
fine, erythematous, macular rash that is most pronounced on his trunk. He
has had a mild fever for 36 hours, but does not appear very sick, and is
happily playing when the pediatrician enters the examining room. Physical
examination demonstrates a reddened throat with tonsillar exudates,
enlarged cervical nodes including the occipital node, and questionable
splenomegaly. The mother says that the boy has not been coughing, and
no Koplik spots are noted. Which of the following is the most likely
diagnosis?
A. Bullous pemphigoid
B. Dermatitis herpetiformis
C. Herpes simplex
D. Measles
E. Rubella



Answer
--------------------------------------------------------------------------------

<8>A child who has had abnormal development of the membranous bones has a
broad skull with associated facial and dental anomalies. Which other bones
are most likely to also be affected?

A. clavicles
B. Femurs
C. Metatarsals
D. Phalanges
E. Tibias

Answer
--------------------------------------------------------------------------------

<9>A 33-year-old woman presents with fever, vomiting, severe irritative
voiding symptoms, and pronounced costovertebral angle tenderness.
Laboratory evaluation reveals leukocytosis with a left shift; blood
cultures indicate bacteremia. Urinalysis shows pyuria, mild hematuria,
and gram-negative bacteria. Which of the following drugs would best
treat this patient's infection?
A. Ampicillin and gentamicin
B. Erythromycin
C. Gentamicin and vancomycin
D. Phenazopyridine and nitrofurantoin
E. Tetracycline

Answer
--------------------------------------------------------------------------------

<10>A surgical pathology specimen from a 24-year-old woman seen at a
reproductive medicine clinic demonstrates a ciliated columnar epithelium.
From which of the following locations in the female genital tract was the
biopsy obtained?
A. Cervix
B. Endometrium
C. Fallopian tube
D. Ovary
E. Vagina

Answer
--------------------------------------------------------------------------------

<11>In a genotypic male, the testes fail to develop, and do not secrete
testosterone or Müllerian regression factor. Which of the following best
describes the in utero reproductive system development of this individual ?
A. Both male- and female-type internal reproductive tracts and male-type external genitalia
B. Female-type internal reproductive tract and female-type external genitalia
C. Female-type internal reproductive tract and male-type external genitalia
D. Male-type internal reproductive tract and female-type external genitalia
E. Male-type internal reproductive tract and male-type external genitalia

Answer
--------------------------------------------------------------------------------

<12>A pulmonologist is testing a patient's lung volumes and capacities using
simple spirometry. Which of the following lung volumes or capacities cannot
be measured directly using this technique?
A. Expiratory reserve volume
B. Functional residual capacity
C. Inspiratory reserve volume
D. Tidal volume
E. Vital capacity

Answer
--------------------------------------------------------------------------------

<13>Contraction of which of the following muscles contributes most to the
backward movement of the lower jaw during the process of mastication?
A. Digastric
B. Lateral pterygoid
C. Medial pterygoid
D. Mylohyoid
E. Temporalis

Answer
--------------------------------------------------------------------------------

<14>A 28-year-old female presents to the doctor complaining of syncopal
episodes that last a few minutes. She is not taking any medications and
has no previous medical history. EEG and EKG studies are performed and
are unremarkable. An echocardiogram shows a single ball-shaped mass
dangling in the left atrium near the mitral valve. The most likely
diagnosis is
A. angiosarcoma
B. mesothelioma
C. myxoma
D. rhabdomyoma
E. rhabdomyosarcoma

Answer
--------------------------------------------------------------------------------

<15>A patient who has been exhibiting various endocrine abnormalities
has an MRI scan of the head. This scan reveals a small tumor of the
pituitary gland. If this tumor expands laterally, which of the following
nerves will most likely be affected first?
A. Abducens nerve
B. Oculomotor nerve
C. Optic nerve
D. Trigeminal nerve
E. Trochlear nerve

Answer
--------------------------------------------------------------------------------

<16>A liver biopsy from a 54-year-old man shows many Mallory bodies. This
finding is most suggestive of which of the following diseases?
A. Alcohol abuse
B. Alpha1-antitrypsin deficiency
C. Hepatitis A
D. Hepatitis B
E. Wilson's disease

Answer
--------------------------------------------------------------------------------

<17>A 27-year-old man has been arrested by the police for hitting, cursing
at, and verbally berating his wife of 8 years. The wife tells the police he
also regularly physically whips his 7-year-old son with a leather belt and
often strikes the boy with his hand. When asked why he does this, he
responds that this is "how my father treated me, it's how men should act." This represents which of the following types of learning?
A. Classical conditioning
B. Cognitive learning
C. Imprinting
D. Operant conditioning
E. Social learning

Answer
--------------------------------------------------------------------------------

<18>A 64-year-old man presents to his doctor with aching, burning pain afte
r meals. He had been self-medicating for several months with antacids, but
he found this to be increasingly ineffective. His physician decides to take
him off the antacids and instead places him on a combination of ranitidine
and sucralfate. Why is this combination a bad idea?
A. Ranitidine increases the toxicity of sucralfate
B. Ranitidine inhibits the action of sucralfate
C. Sucralfate and ranitidine coprecipitate
D. Sucralfate increases the toxicity of ranitidine
E. Sucralfate inhibits the action of ranitidine

Answer
--------------------------------------------------------------------------------

<19>A 42-year-old female presents with a recent onset of fatigue, malaise,
constipation, and a 12-pound weight gain. On examination, her thyroid is
firm and enlarged. What laboratory test is most likely to confirm the
expected diagnosis?
A. Antithyroid antibodies
B. Serum thyroid-stimulating hormone (TSH) measurement
C. Serum thyroxine (T4) measurement
D. Serum triiodothyronine (T3) measurement
E. T3 resin uptake

Answer
--------------------------------------------------------------------------------

<20>A 70-year-old man with a history of atrial fibrillation is started on an
oral anticoagulant. His prothrombin time is monitored on a regular basis. A
few months into his therapy, he begins treatment for a duodenal ulcer and
he develops symptoms of a bleeding diathesis. Which of the following ulcer
medications is most likely responsible for this change in his hemostatic
status?
A. Cimetidine
B. Famotidine
C. Misoprostol
D. Omeprazole
E. Ranitidine

Answer
--------------------------------------------------------------------------------

<21>An elderly woman with a history of multiple oral ulcers presents with
flaccid bullae on her scalp, face, and trunk. Manual pressure on the skin
produces separation of the upper layer of the epidermis, followed by
eventual sloughing of the skin. The patient has been in relatively good
health until recently, and denies taking any medications. A biopsy of one
of the skin lesions reveals separation of epithelial cells above the
basal layer. Autoantibodies to which of the following components would
most likely be found in this patient?
A. Epidermal basement membrane proteins
B. Glycoprotein IIb/IIIa
C. Intercellular junctions of epidermal cells
D. Intrinsic factor
E. Type IV collagen

Answer
--------------------------------------------------------------------------------

<22>A 35-year-old woman notices a change in the appearance of a mole on her
neck. Physical examination reveals that the lesion is an irregular, nodular,
superficial mass with a variegated appearance. Biopsy demonstrates a primary malignant tumor. Which of the following factors is most predictive of the patient's long term prognosis?
A. Circumference of lesion
B. Darkness of lesion
C. Degree of color variation
D. Depth of lesion
E. Sharpness of border between lesion and adjacent skin

Answer
--------------------------------------------------------------------------------

<23>During embryological development, hematopoiesis occurs in different
organs at different times. Which of the following are the correct organs,
in the correct sequence, at which hematopoiesis occurs embryologically?
A. Amnion, yolk sac, placenta, bone marrow
B. Placenta, liver and spleen, yolk sac, bone marrow
C. Placenta, spleen and lymphatic organs, bone marrow
D. Yolk sac, bone marrow, liver and spleen
E. Yolk sac, liver, spleen and lymphatic organs, bone marrow

Answer
--------------------------------------------------------------------------------

<24>A 6-month-old boy is brought to the pediatrician by his parents, who are
first cousins. This is their first child. Physical examination reveals a
small, thin, lethargic infant with slightly misshapen long bones. His
features are somewhat coarse. Joint movements are restricted, his corneas
are clouded, and his gums are underdeveloped. His liver is not enlarged.
Serum levels of acid hydrolases are found to be elevated. The child most
likely has a defect in which of the following metabolic activities?
A. Degradation of dermatan sulfate and heparan sulfate
B. Degradation of gangliosides
C. Degradation of glycogen
D. Degradation of sphingomyelin
E. Phosphorylation of mannose moieties
F. Phosphorylation of tyrosine moieties

Answer
--------------------------------------------------------------------------------

<25>A particular association determines membership on the basis of members'
IQ scores. Only those persons who have documented IQ scores at least 2
standard deviations above the mean on the Wechsler Adult Intelligence
Scale (WAIS) are eligible for admission. Of a group of 200 people randomly
selected from the population at large, how many would be eligible for
membership to this society?
A. 1
B. 2
C. 3
D. 4
E. 5

Answer
--------------------------------------------------------------------------------

<26>5 mL of synovial fluid is aspirated from an inflamed knee joint. The
fluid is yellow-white and cloudy and contains
200,000 WBC/mm3 (85% neutrophils). Needle-shaped,
strongly negatively birefringent crystals are seen both within
and outside neutrophils. These crystals most likely have which of
the following compositions?
A. Basic calcium phosphate
B. Calcium oxalate
C. Calcium pyrophosphate dihydrate
D. Cholesterol
E. Monosodium urate

Answer
--------------------------------------------------------------------------------

<27>A patient has a painful ulcer on the tip of his tongue. Which of the
following cranial nerves carries the pain sensation he experiences?
A. V2
B. V3
C. VII
D. IX
E. X

Answer
--------------------------------------------------------------------------------

<28>A 30-year-old veterinarian on a cattle ranch presents with a 1-to-2-month
history of malaise, chills, drenching malodorous sweats, fatigue, and
weakness. He has anorexia and has lost 15 pounds. He has intermittent
fevers that range up to 103 F (39.4 C). He complains of visual blurring.
A physical examination reveals mild lymphadenopathy, petechiae, and a
cardiac murmur consistent with aortic insufficiency. What is the most
likely etiologic agent?
A. Bacillus anthracis
B. Brucella abortus
C. Coccidioides immitis
D. Erysipelothrix rhusiopathiae
E. Trichinella spiralis

Answer
--------------------------------------------------------------------------------

<29>The parents of a 6-month-old child who was normal at birth bring her
into the clinic. Since their emigration to the U.S. from Eastern Europe
soon after her birth, the child has developed diminished responsiveness
, progressive blindness and deafness, and recently, seizures. Serum levels
of which of the following compounds would be expected to be decreased in
both of the parents?
A. Dystrophin
B. Hexosaminidase A
C. Hypoxanthine-guanine phosphoribosyltransferase (HGPRT)
D. Phenylalanine hydroxylase
E. Vitamin D3

Answer
--------------------------------------------------------------------------------

<30>A 60-year-old male with angina comes to the emergency room with severe
chest pain unresponsive to sublingual nitroglycerin. An EKG shows ST
segment elevation in the anterolateral leads, and thrombolytic therapy
is initiated. If streptokinase is given to this patient, it may produce
thrombolysis after binding to which of the following proteins?
A. Antithrombin III
B. Fibrin
C. Plasminogen
D. Protein C
E. Thrombomodulin

Answer
--------------------------------------------------------------------------------

<31>The left adrenal vein drains directly into which of the following veins?
A. Hemiazygos vein
B. Inferior vena cava
C. Left renal vein
D. Splenic vein
E. Superior mesenteric vein

Answer
--------------------------------------------------------------------------------

<32>A premature infant develops progressive difficulty breathing over the
first few days of life. Deficient surfactant synthesis by which of the
following cell types may have contributed to the baby's respiratory problems?
A. Alveolar capillary endothelial cells
B. Bronchial mucous cells
C. Bronchial respiratory epithelium
D. Type I pneumocytes
E. Type II pneumocytes

Answer
--------------------------------------------------------------------------------

<33>A surgeon performs an exploratory laparotomy, producing a large incision
in the patient's abdomen. Poor blood supply to which of the following is
most likely to cause problems during the healing process?
A. Adipose tissue
B. Aponeuroses
C. Loose connective tissue
D. Muscle
E. Skin

Answer
--------------------------------------------------------------------------------

<34>A 1-year-old child develops voluminous watery diarrhea and vomiting. She
is brought to the pediatrician by her parents and evaluated, then sent home
with instructions for the parents to give the child an electrolyte
replacement solution. Which of the following viruses is the most likely
cause of the child's diarrhea?
A. Coronavirus
B. Lymphocytic choriomeningitis virus
C. Norwalk agent
D. Orbivirus
E. Rotavirus

<35>A 2-year-old child presents to the pediatrician with hematuria.
Examination reveals hypertension and an abdominal mass. A tumor is localized
to the right kidney and biopsy reveals a stroma containing smooth and
striated muscle, bone, cartilage, and fat, with areas of necrosis.
The gene for this disorder has been localized to which of the following
chromosomes?
A. 5
B. 11
C. 13
D. 17
E. 22
Answer
--------------------------------------------------------------------------------

<36>A newborn baby has a prominent defect at the base of his spine through
which his meninges and spinal cord protrude. A failure of which of the
following processes is the most common cause of this type of defect?
A. Development of primary vertebral ossification centers
B. Development of the body
C. Development of the pedicle
D. Development of the superior articular process
E. Fusion of the vertebral arches

Answer
--------------------------------------------------------------------------------

<37>A 45-year-old male presents to the physician with muscle cramps,
perioral numbness, and irritability over the past 3 to 4 months. Lab
results reveal hypocalcemia, normal albumin level, and hyperphosphatemia
. Parathyroid hormone level is decreased. Alkaline phosphatase level is
normal. Which of the following is most likely causing this clinical
scenario?
A. Bone metastases
B. Hashimoto's thyroiditis
C. Hypervitaminosis D
D. Hypomagnesemia
E. Previous subtotal thyroidectomy

Answer
--------------------------------------------------------------------------------

<38>In which of the following organs are fenestrated endothelial cells
common?
A. Heart
B. Liver
C. Lungs
D. Pancreas
E. Stomach

Answer
--------------------------------------------------------------------------------

<39>A macroscopic hepatic change known as nutmeg liver is indicative of
A. acute left-sided heart failure
B. acute right-sided heart failure
C. alcohol toxicity
D. chronic congestive heart failure
E. liver cirrhosis

Answer
--------------------------------------------------------------------------------

<40>A 26-year-old man is admitted through the emergency department to the
hospital for a heroin overdose. His heart rate is 45 beats/min, and his
blood pressure is 75/40 mm Hg. Which of the following best depicts the
results from an arterial blood sample ?
pH PaCO2 (mm Hg) HCO3- (mEq/L)
A. 7.22 66 26
B. 7.34 29 15
C. 7.40 40 24
D. 7.47 20 14
E. 7.49 48 35


Answer
--------------------------------------------------------------------------------

<41> The maximum expiratory flow-volume curve shown above is created when
the patient inhales as much air as possible and then expires with maximum
effort until no more air can be expired. What is the forced vital capacity
of this patient?
A. 1.5 Liters
B. 2.5 Liters
C. 3.5 Liters
D. 4.5 Liters
E. 6.0 Liters

Answer
--------------------------------------------------------------------------------

<42>A 25-year-old male gets into a brawl outside a bar. During the
altercation, someone pulls out a gun and shoots him in the head.
The bullet enters the man's temple and severs his right optic nerve
completely. He is quickly transported to a nearby emergency room and
an emergency physician tests his pupillary response by shining a light
in the right eye. What will the physician most likely find?
A. No pupillary constriction in the right eye, and no pupillary constriction in the left eye
B. No pupillary constriction in the right eye, but pupillary constriction in the left eye
C. Pupillary constriction followed by pupillary dilatation in both eyes
D. Pupillary constriction in the right eye, and no pupillary constriction in the left eye
E. Pupillary constriction in both eyes
F. Pupillary dilatation in both eyes

Answer
--------------------------------------------------------------------------------

<43>A patient complaining of chest pain with exercise is evaluated by
cardiac catheterization. The left anterior descending (LAD) branch of
the coronary artery is visualized but the contrast angiography is poor
. A Doppler-tipped catheter is inserted and the blood velocity is observed
to increase transiently from 10 cm/sec to 70 cm/sec and then decrease bac
k to 10 cm/sec as the probe passes a particular location in the artery
. What was the cause of these changes in velocity measurements?
A. A coronary artery aneurysm with a cross-sectional area 1/7th the size of the native artery
B. A coronary artery aneurysm with a cross-sectional area 7 times greater than the native artery
C. A coronary artery obstruction with a cross-sectional area 1/7th of the size of the native artery
D. A coronary artery obstruction with a cross-sectional area 7 times greater than the native artery


Answer
--------------------------------------------------------------------------------


<44>Type of blood vessel Fall in blood pressure (mm Hg) (% of total peripheral resistance)
Aorta and large arteries <1
Small arteries 10-20
Arterioles 50
Capillaries 25
Venules and small veins 9
Vena cave <1

The table above shows the fall in blood pressure that occurs for the
various types of blood vessels as blood flows from the aorta (100 mm Hg)
to the right atrium (0 mm Hg). Which of the following types of blood
vessel is likely to have the highest ratio of wall cross-sectional
area to lumen cross-sectional area?
A. Aorta and large arteries
B. Small arteries
C. Arterioles
D. Capillaries
E. Venules and small veins
F. Vena cavae

Answer
--------------------------------------------------------------------------------

<45>Evaluation of an infant with a variety of congenital abnormalities
reveals hypocalcemia due to a lack of parathyroid hormone. On x-ray, the
thymic shadow is absent. A failure of development and differentiation of
which of the following embryonic structures would most likely be
responsible for the observed presentation?
A. Second pharyngeal arch
B. Second pharyngeal cleft
C. Second pharyngeal pouch
D. Third pharyngeal arch
E. Third pharyngeal pouch

Answer
--------------------------------------------------------------------------------

<46>A person lifts one foot prior to taking a step. Which of the following
nerves innervates the muscle group that allows the person to maintain
balance by holding the weight of his body over the foot remaining on
the ground?
A. Femoral nerve
B. First and second sacral nerves
C. Obturator nerve
D. Superior gluteal nerve
E. Tibial nerve
Answer
--------------------------------------------------------------------------------
<47>A 21-year-old college student from Connecticut with a past history
of Lyme disease presents with chronic pain and swelling in his right
knee. He states that he has had problems with the knee for the past
two years. Which of the following HLA alleles would you expect to
be present in this individual?
A. HLA-B9
B. HLA-B17
C. HLA-B27
D. HLA-DR3
E. HLA-DR4
Answer
--------------------------------------------------------------------------------

<48>Which of the following metabolic processes occurs exclusively in
the mitochondria?
A. Cholesterol synthesis
B. Fatty acid synthesis
C. Gluconeogenesis
D. Glycolysis
E. Hexose monophosphate shunt
F. Ketone body synthesis
G. Urea cycle


Answer
--------------------------------------------------------------------------------

49>An alert pediatric intern notices that a neonate with dysmorphic facies
is twitching abnormally. As he watches, the baby experiences a seizure.
Stat laboratories indicate a glucose of 90, serum sodium of 140, serum
potassium of 4.2 and serum calcium of 3.9. Over the next several months,
the child is admitted to the hospital twice for Candida infections, and
once for a viral exanthem. Which of the following is the most likely
diagnosis?
A. Ataxia telangiectasia
B. Bruton's hypogammaglobulinemia
C. DiGeorge syndrome
D. Severe combined immunodeficiency
E. Wiskott-Aldrich syndrome
Answer
--------------------------------------------------------------------------------

50>A 2-month-old boy is evaluated for failure to thrive. As the pediatrician
is examining the patient, she witnesses a seizure. Physical examination
is remarkable for hepatomegaly, a finding later confirmed by CT scan,
which also reveals renomegaly. Serum chemistries demonstrate severe
hypoglycemia, hyperlipidemia, lactic acidosis, and ketosis. Which of
the following diseases best accounts for this presentation?
A. Gaucher's disease
B. McArdle's disease
C. Niemann-Pick disease
D. Pompe's disease
E. von Gierke's disease
Answer
--------------------------------------------------------------------------------
Answer
--------------------------------------------------------------------------------

Answers
--------------------------------------------------------------------------------
--------------------------------------------------------------------------------
--------------------------------------------------------------------------------
--------------------------------------------------------------------------------
1>The correct answer is B. One of the most common side effects of any antineoplastic therapy is weight loss secondary to decreased appetite and/or nausea and vomiting. Furthermore, weight loss due to decreased food intake tends to occur more frequently in elderly patients receiving antineoplastic therapy. One medication that has consistently helped to increase appetite in such patients is megestrol acetate. This agent is a progestational hormone with antineoplastic properties used in the treatment of advanced carcinoma of the breast and endometrium. Megestrol, when given in relatively high doses, can substantially increase the appetite in most individuals, even those with advanced cancer.
Amitriptyline (choice A) is a tricyclic antidepressant used in the treatment of depression. There is nothing mentioned in the case study to suggest that the patient is clinically depressed; hence, this agent would provide no benefit.
Methotrexate (choice C) is an antimetabolite and folic acid antagonist commonly used in various neoplastic disorders and in the treatment of rheumatoid arthritis. Since nausea, vomiting, and ulcerative stomatitis are common side effects of this medication, its usage in this patient would not be recommended.
Neostigmine (choice D) is a carbamylating acetylcholinesterase inhibitor that would not increase appetite.
Prochlorperazine (choice E) is a phenothiazine derivative used primarily to control severe nausea and vomiting. This patient is not experiencing nausea. Furthermore, this agent does not possess appetite-stimulating properties.
--------------------------------------------------------------------------------
2>The correct answer is B. The principle host defense in mycobacterial infections (such as this patient's tuberculosis) is cell-mediated immunity, which causes formation of granulomas. Unfortunately, in tuberculosis and in many other infectious diseases characterized by granuloma formation, the organisms may persist intracellularly for years in the granulomas, only to be a source of activation of the infection up to decades later.
While antibody-mediated phagocytosis (choice A) is a major host defense against many bacteria, it is not the principle defense against Mycobacteria.
IgA-mediated hypersensitivity (choice C) is not involved in the body's defense against Mycobacteria.
IgE-mediated hypersensitivity (choice D) is not involved in the body's defense against Mycobacteria. It is important in allergic reactions.
Neutrophil ingestion of bacteria (choice E) is a major host defense against bacteria, but is not the principle defense against Mycobacteria.
--------------------------------------------------------------------------------
3>The correct answer is E. The immunoperoxidase method uses horseradish peroxidase to produce a visible pigment when a specific antibody binds to antigenic sites in the tissue. Binding of the antibody to the epithelial cells indicates that the protein in question is being expressed by those cells. The positive immunoperoxidase results obtained here suggests that the channel protein is expressed in proximal convoluted tubular epithelium, since this is the only site in the kidney at which the epithelial cells have a "brush border." The brush border is made of microvilli, which enhance the proximal tubules' ability to reabsorb plasma constituents filtered at the glomeruli.
--------------------------------------------------------------------------------
4>The correct answer is D. Pregnant women with chronic hypertension "require" antihypertensive therapy when the diastolic pressure is greater than 100 mm Hg; however, some clinicians may decide to treat patients with diastolic blood pressures less than 100 mm Hg. For the initiation of therapy, methyldopa is still considered to be the agent of choice. Methyldopa is converted intraneuronally to a-methylnorepinephrine, an alpha-2 adrenergic agonist, which is subsequently released. Release of a-methylnorepinephrine in the medulla leads to a decrease in sympathetic outflow, thus lowering blood pressure. Methyldopa has been safely used in the treatment of hypertension during pregnancy; this agent is not associated with the development of teratogenic or other fetal abnormalities.
Diuretics, such as bumetanide (choice A) and hydrochlorothiazide (choice C), are often avoided since these agents can produce hypovolemia, leading to reduced uterine blood flow. Although these agents can be used during pregnancy, methyldopa and hydralazine are the drugs of choice for hypertension during pregnancy.
Fosinopril (choice B) is an angiotensin-converting enzyme (ACE) inhibitor that should not be administered to pregnant women, especially in the second or third trimesters. These agents have been associated with severe fetal and neonatal injury, such as hypotension, neonatal skull hypoplasia, anuria, renal failure, and death.
Along the same lines, the use of the angiotensin II receptor antagonists, such as valsartan (choice E), is not recommended since these agents cause fetal complications similar to the ACE inhibitors.
--------------------------------------------------------------------------------
5>The correct answer is D. The disease is Duchenne muscular dystrophy, an X-linked recessive muscular disease usually caused by a deletion involving the dystrophin gene. This defect produces accelerated muscle breakdown leading initially to proximal muscle weakness, then later to generalized weakness that typically begins before age 5. A feature of X-linked recessive diseases is that carrier mothers pass the disease to half their sons; affected fathers can have carrier daughters but not affected sons. Since the mother is presumably normal (because the disease is X-linked), she must be a carrier to have an affected son, and the grandmother must also be a carrier, therefore the mother's brother (maternal uncle) may also have the disease.
The father's (choice A) side of the family, including the father's brother (choice B), most likely does not carry the defective gene (since they themselves would be affected, and furthermore since the father cannot pass the gene on to a son). It would be extremely unlikely for a carrier female to marry an affected male (and the question does not mention any similar symptoms in the father).
The mother (choice C) and possibly the sister (choice E) are carriers of, but not affected by, the defective gene.
--------------------------------------------------------------------------------
6>The correct answer is B. The cell pictured is an eosinophil, a member of the granulocytic lineage of white blood cells. The crystalline core of the granule contains a protein called the major basic protein, which appears to function in the destruction of parasites. Major basic protein also has deleterious effects on epithelial cells in patients with asthmatic reactions. The light component around the dense crystalline core contains products such as histaminase, arylsulfatase, and other enzymes.
Lactoferrin (choice A) is found in the specific granules of the neutrophil. It inhibits the growth of bacteria by interfering with iron metabolism.
Myeloperoxidase (choice C) is found in the azurophilic (large) granule of the neutrophil. This enzyme is also destructive to bacteria, destroying their cell walls.
Histamine (choice D) is produced by the basophil and the mast cell. The histaminase of the eosinophil regulates the inflammatory reaction of these two cell types.
Tartrate-resistant acid phosphatase (choice E) is a marker for hairy cell leukemia, a neoplasm of the B lymphocyte line.
--------------------------------------------------------------------------------
7>The correct answer is E. This presentation (fine, nonblotchy, truncal rash in a not-very-ill child) is characteristic of rubella, or German measles. IgM specific for rubella can often be detected in serum within 1-2 days of developing the rash. The principal significance of this disease is that it can cause a devastating congenital infection characterized by ocular problems (cataracts, retinopathy, microphthalmos, glaucoma), cardiovascular problems (patent ductus arteriosus, ventricular septal defect, pulmonary stenosis), deafness, thrombocytopenic purpura, hepatosplenomegaly, CNS problems, and bony lesions.
Bullous pemphigoid (choice A) produces large, tense blisters.
Dermatitis herpetiformis (choice B) causes recurrent crops of small vesicles or papules.
Herpes simplex (choice C) is characterized by crops of vesicles on oral or genital sites.
Measles (choice D) causes a blotchy, maculopapular erythematous rash that begins on the face and spreads downward. Patients with measles are usually much sicker than those with German measles, and Koplik spots maybe seen on the buccal mucosa.
--------------------------------------------------------------------------------
8>The correct answer is A. In a syndrome called cleidocranial dysostosis, absence of part of the clavicles accompanies a broad skull, and facial and dental anomalies. Note that you could also have answered this question by noting that of the bones listed, only the clavicles form by intramembranous ossification.
The femurs (choice B), metatarsals (choice C), phalanges (choice D), and tibias (choice E) are cartilaginous (formed by endochondral ossification) rather than membranous bones.
--------------------------------------------------------------------------------
9>The correct answer is A. Acute pyelonephritis is an infectious disease involving the kidney parenchyma and the renal pelvis. Gram-negative bacteria, such as Escherichia coli, Proteus, Klebsiella, and Enterobacter, are the most common causative organisms in acute pyelonephritis. Laboratory evaluation will often reveal leukocytosis with a left shift, and urinalysis typically shows pyuria, varying degrees of hematuria, and white cell casts. Since bacteremia is present, the patient should be hospitalized and empirically started on IV ampicillin and gentamicin. This regimen may be need to be changed, however, once the sensitivity results are available.
Erythromycin (choice B) and tetracycline (choice E) are both bacteriostatic antibiotics and would not be recommended in a patient with a severe infection, such as acute pyelonephritis with bacteremia.
Vancomycin (choice C) is primarily used in the treatment of severe gram-positive infections.
Phenazopyridine (choice D) is a urinary analgesic, and nitrofurantoin (choice D) is a urinary tract anti-infective. Although nitrofurantoin is indicated for the treatment of "mild" cases of pyelonephritis, as well as cystitis, this patient's condition is severe and should be treated with appropriate antibiotics.
--------------------------------------------------------------------------------
10>The correct answer is C. The fallopian tube is the only structure in the female genital tract with a ciliated columnar epithelium; the beating of the cilia helps move the egg into the uterus. This fact is also sometimes clinically helpful since dilated and deformed fallopian tubes can be microscopically distinguished from cystic ovarian tumors by the presence of the cilia.
The cervix (choice A) and vagina (choice E) are lined by squamous epithelium.
The endometrium (choice B) is lined by columnar epithelium (although a few ciliated cells may be present).
The covering of the ovary (choice D) is cuboidal epithelium, and cysts within the ovary can be lined by cuboidal or non-ciliated columnar epithelium.
--------------------------------------------------------------------------------
11>The correct answer is B. The description is that of gonadal dysgenesis. In the absence of testosterone, the Wolffian ducts will regress and fail to differentiate into normal male internal reproductive tracts. In the absence of Müllerian regression factor, the Müllerian ducts will automatically differentiate into oviducts and a uterus. Differentiation of the male external genitals is dependent on adequate dihydrotestosterone (via an action of 5 a-reductase on testosterone). In the absence of testosterone, female-type external genitalia will develop.
Selective dysgenesis of the Sertoli cells could produce the situation described in choice A. Normal Leydig cells would secrete testosterone and produce normal male-type internal and external tracts. However, the absence of Müllerian regression factor, which is secreted by the Sertoli cells, would allow formation of female-type internal structures as well.
Female-type internal reproductive tract and male-type external genitalia (choice C) would not be likely to occur under any circumstances.
The situation described in choice D could occur with 5 a-reductase deficiency. Normal male-type internal tracts can form because there is no requirement for dihydrotestosterone. Müllerian regression factor will prevent differentiation of female-type internal tracts. Since differentiation of the normal male external genitals requires dihydrotestosterone, 5 a-reductase deficiency will lead to feminization.
The situation described in choice E is normal, and would not occur in the individual described who has testicular dysgenesis.
--------------------------------------------------------------------------------
12>The correct answer is B. The functional residual capacity is the amount of air left in the lungs after a normal expiration. Because this volume cannot be expired in its entirety, it cannot be measured by spirometry. Essentially, lung volume that contains the residual volume, which is the amount of air remaining after maximal expiration (e.g., functional residual capacity and total lung capacity), cannot be measured by spirometry. These volumes can be determined using helium dilution techniques coupled with spirometry or body plethysmography.
The expiratory reserve volume (choice A) is the volume of air that can be expired after expiration of a tidal volume.
The inspiratory reserve volume (choice C) is the volume of air that can be inspired after inspiration of a tidal volume.
Tidal volume (choice D) is the amount of air inspired or expired with each normal breath.
Vital capacity (choice E) is the volume of air expired after a maximal inspiration
--------------------------------------------------------------------------------
13>The correct answer is E. Mastication is a complex process involving alternating elevation, depression, forward movement, and backward movement of the lower jaw. The backward movement step is accomplished by the posterior fibers of the temporalis muscle.
The digastric (choice A) helps to depress the lower jaw during chewing.
The lateral pterygoid (choice B) helps to move the lower jaw forward during chewing.
The medial pterygoid (choice C) helps to elevate the lower jaw during chewing.
The mylohyoid (choice D) helps to depress the lower jaw during chewing.
--------------------------------------------------------------------------------
14>The correct answer is C. The vignette illustrates a typical presentation for a tumor of the heart. Primary cardiac tumors are rare and usually require an intensive work-up to pinpoint the diagnosis. 75% of primary cardiac tumors are benign and among these, myxoma is the most common. The tumors are usually single; the most common location is the left atrium. They may cause syncopal episodes or even shock and death due to obstruction by a "ball valve" mechanism.
Angiosarcoma (choice A) is a malignant tumor of vascular origin that can occur as a primary cardiac tumor. It is the most common malignant primary cardiac tumor, but it is still very rare. Angiosarcoma usually affects the right side of the heart.
Mesothelioma (choice B) is a benign tumor of mesothelial origin that can rarely present as a primary cardiac tumor. It is usually a small intramyocardial tumor that presents with disturbances of the conduction system of the heart.
Rhabdomyoma (choice D) is a benign tumor of muscle origin. It can occur as a primary cardiac tumor, typically in infants and children, in whom it may be associated with tuberous sclerosis. It usually occurs in the ventricles.
Rhabdomyosarcoma (choice E) is a malignant neoplasm that can also occur as a rare primary cardiac tumor. It is of muscle origin and usually affects the right heart.
--------------------------------------------------------------------------------
15>The correct answer is A. The pituitary gland is located in the pituitary fossa within the skull. The floor of this fossa is formed by the sella turcica. The lateral walls of the fossa are formed by the cavernous sinuses. The abducens nerve passes through the cavernous sinus along with the internal carotid artery. As the tumor expands laterally, the first nerve that will be encountered is the abducens nerve, producing a lateral rectus palsy.
The oculomotor nerve (choice B) lies in the lateral wall of the cavernous sinus. It is further from the pituitary gland than is the abducens nerve.
The optic nerve (choice C) is anterosuperior to the pituitary gland. Upward expansion of the tumor may compress the optic chiasm.
The trigeminal nerve (choice D) is found posterior to the cavernous sinus. Two of its three divisions (ophthalmic and maxillary divisions) pass through the lateral wall of the cavernous sinus and are further from the pituitary gland than is the abducens nerve.
The trochlear nerve (choice E) is also in the lateral wall of the cavernous sinus, and would be affected later if the tumor continued to expand.
--------------------------------------------------------------------------------
16>The correct answer is A. Mallory bodies are eosinophilic cytoplasmic inclusions ("alcoholic hyaline") that are found in the largest numbers in alcoholic hepatitis. They were originally considered to be pathognomic of alcohol abuse, but have since been found (in much smaller numbers) in many other liver conditions.
Alpha1-antitrypsin deficiency (choice B) involvement of the liver is characterized by periodic acid Schiff (PAS)-positive cytoplasmic granules in hepatocytes.
Hepatitis A (choice C) and hepatitis B (choice D) infections are definitively established with serologic markers.
In Wilson's disease (choice E), there is excess copper deposition in the liver
--------------------------------------------------------------------------------
17>The correct answer is E. In social learning, also known as modeling, behavior is acquired by watching other persons and assimilating their actions into the behavioral repertoire. There is no verbal or cognitive process (choice B) that is involved, no reinforcement (as in operant conditioning; choice D), no pairing of stimuli to get stimulus substitution (as in classical conditioning; choice A), nor any early-life bonding or imprinting (choice C) involved in this type of process. Because behaviors such as spousal abuse, child abuse, and elder abuse are all based on observing and incorporating behaviors from significant others, the person displaying the behaviors does not realize the behaviors are inappropriate and is typically very resistant to change. The fact that the learning is nonverbal and not dependent upon reinforcement contributes to the resistance to change.
--------------------------------------------------------------------------------
18>The correct answer is B. Sucralfate is a promising drug that is not presently in widespread use because it is incompatible with H2 antagonists such as cimetidine, ranitidine, famotidine and nizatidine. Sucralfate is aluminum sucrose sulfate, a sulfated disaccharide, which polymerizes and binds to ulcerated tissue. It forms a protective coating against acid, pepsin and bile, giving the tissue a chance to heal. Unfortunately, a low gastric pH is required for polymerization, meaning that sucralfate is incompatible with drugs that reduce gastric acidity, such as H2 blockers and antacids. The moral of the story is that you cannot assume that two medications that are individually helpful in a medical condition will be synergistic. Learning the mechanisms by which the drugs work will help you spot potential interactions and earn you points on the USMLE.
--------------------------------------------------------------------------------
19>The correct answer is B. The patient's presentation is consistent with hypothyroidism. Serum thyroid-stimulating hormone (TSH) measurement (choice B) is most likely to confirm the empiric diagnosis. TSH levels usually rise above normal before serum thyroxine (T4; choice C) and serum triiodothyronine (T3; choice D) levels do, even in mild cases of hypothyroidism. Therefore, TSH measurement would be the most accurate test to determine the presence of hypothyroidism regardless of the severity.
A high titer of antithyroid antibodies (choice A) is characteristic of chronic thyroiditis, which is the most common cause of hypothyroidism. However, detection of these antibodies would not indicate if hypothyroidism was present.
T3 resin uptake (choice E) measurement is not an accurate test of thyroid function; it is primarily used to exclude various abnormalities in the thyroid-hormone binding proteins.
--------------------------------------------------------------------------------
20>The correct answer is A. Warfarin is the oral anticoagulant the patient was most likely taking. This drug is commonly prescribed to patients with atrial fibrillation to prevent the formation of atrial thrombi. Warfarin increases prothrombin time (PT) because it interferes with the synthesis of the vitamin K clotting factors of the liver (II, VII, IX, and X) and therefore necessitates regular monitoring of the PT. Cimetidine is an H2-blocker that inhibits hepatic enzymes, including those that metabolize warfarin. Consequently, coadministration of warfarin and cimetidine results in enhanced warfarin activity, producing pronounced anticoagulation and the bleeding diathesis in the patient in question. Cimetidine has one of the worst side effect profiles of all the H2-blockers and may also result in gynecomastia in men.
Famotidine (choice B) is an H2-blocker that does not affect liver metabolism.
Misoprostol (choice C) is a prostaglandin E1 analog used in peptic ulcer disease. It does not affect hepatic metabolism.
Omeprazole (choice D) is a proton-pump inhibitor used to decrease acid production in patients with peptic ulcer disease or reflux. It does not affect drug metabolism by the liver.
Ranitidine (choice E) is another H2-blocker. It does not inhibit liver enzymes as strongly as cimetidine does.
--------------------------------------------------------------------------------
21>The correct answer is C. Bullae with the cleavage plane above the basal layer of the epidermis suggests pemphigus vulgaris, which is caused by autoantibodies to intercellular junctions of epidermal cells. The autoantibodies decrease the ability of the keratinocytes to adhere to one another, permitting formation of vesicles and bullae. Oral involvement is common, and often precedes the characteristic skin lesions. Separation of the epidermis upon manual stroking of the skin is known as Nikolsky's sign. This sign is present in other disorders such as Stevens-Johnson syndrome, but we are told the woman is not taking any medications, a typical cause of Stevens-Johnson syndrome in the adult population.
Antibodies to epidermal basement membrane proteins (choice A) are seen in bullous pemphigoid, which is a bullous disease characterized by blisters with a cleavage line between the epidermis and dermis.
Antibodies to glycoprotein IIb/IIIa (choice B) are seen in autoimmune thrombocytopenic purpura.
Antibodies to intrinsic factor (choice D) are seen in pernicious anemia.
Antibodies to Type IV collagen (choice E) are seen in Goodpasture's syndrome.
--------------------------------------------------------------------------------
22>The correct answer is D. The lesion is a malignant melanoma. Melanomas can develop either de novo or in an existing mole. Sunlight exposure is a significant risk factor and fair-skinned persons are at increased risk of developing melanoma. The most significant factor for long term prognosis is the depth of the lesion, since the superficial dermis lies about 1 mm under the skin surface, and penetration to this depth is associated with a much higher incidence of metastasis than is seen with a more superficial location.
The circumference of the lesion (choice A) is much less important than depth, since one form of melanoma (superficial spreading) can still have good prognosis despite large size, if it has not extended to the depth of the superficial dermal lymphatic bed.
The darkness (choice B) or degree of variation in color (choice C) do not have prognostic significance once melanoma is diagnosed.
Irregularity, or fuzziness at the border (choice E) of a mole-like lesion is a good clue to potential malignancy, but does not affect prognosis once a melanoma is diagnosed.
--------------------------------------------------------------------------------
23>The correct answer is E. By the third week of development, hematopoiesis begins in the blood islands of the yolk sac. Beginning at 1 month of age and continuing until 7 months of age, blood elements are also formed in the liver. Hematopoiesis occurs in the spleen and lymphatic organs between 2 and 4 months, and in the bone marrow after 4 months.
--------------------------------------------------------------------------------
24>The correct answer is E. The patient has I-cell disease, also known as mucolipidosis II, which is due to a defective UDP-N-acetylglucosamine-1-phosphotransferase, the enzyme that phosphorylates mannose on enzymes destined for lysosomes. Proteins coded by nuclear DNA are synthesized on cytoplasmic ribosomes, which may be either "free" or associated with the endoplasmic reticulum to form the rough endoplastic reticulum (RER). Proteins synthesized on the RER are transferred into the Golgi apparatus, where they undergo further modifications that determine whether they remain part of the Golgi apparatus, become part of the plasma membrane, or are shipped to lysosomes or mitochondria. Proteins not marked for transport to a specific intracellular site follow the default pathway and are exported into the extracellular compartment. The signal for transport of the acid hydrolases (and probably other enzymes) to the lysosomes is phosphorylation of a terminal mannose moiety on an N-linked oligosaccharide to form mannose 6-phosphate. In I-cell disease, this terminal mannose moiety is not phosphorylated, and the acid hydrolases follow the default pathway and are secreted.
Deficiency of alpha-L-iduronidase results in lysosomal accumulation of dermatan sulfate and heparan sulfate (choice A) in several conditions such as mucopolysaccharidosis I, Hurler's disease, or Hurler's/Scheie disease.
Hexosaminidase A deficiency (Tay-Sachs disease) is one example of a condition in which ganglioside accumulation occurs (choice B).
There are a number of diseases in which glycogen degradation (choice C) is defective. These are collectively termed glycogen storage diseases since they result in abnormal cellular accumulation of glycogen. In Pompe's disease, or type II glycogen storage disease, a lysosomal glucosidase is deficient, resulting in lysosomal glycogen accumulation.
Deficiency of sphingomyelinase (choice D), an enzyme involved in degradation of sphingomyelin, results in Niemann-Pick disease.
Phosphorylation of tyrosine moieties (choice F) is unrelated to lysosomes or lysosomal enzymes; however, decreased ability to phosphorylate tyrosine moieties might be associated with diabetes or dwarfism.
--------------------------------------------------------------------------------
25>The correct answer is E. 95% of a normally distributed population will fall between plus or minus 1.96 standard deviations from the mean. Since the population is normally distributed with regard to IQ, this means that approximately 2.5% of the population will have IQ scores 2 standard deviations or more above the mean, and 2.5% of the population will have IQ scores 2 standard deviations or more below the mean. 2.5% of 200 people is 5 people.
--------------------------------------------------------------------------------
26>The correct answer is E. All the compounds listed can produce crystals in joint fluid, but only monosodium urate (associated with gout) and calcium pyrophosphate dihydrate (associated with CPPD crystal deposition disease, also called pseudogout), and to lesser degree basic calcium phosphate (apatite-associated arthropathy), have a high likelihood of being encountered on a step 1 USMLE exam. The crystals described are those of monosodium urate. Be careful not to answer "uric acid" if that is listed as an alternative choice on an exam, since the sodium salt is the predominant species in vivo.
Basic calcium phosphate (choice A) is seen in apatite-associated arthropathy and produces spherical clumps of nonbirefringent submicroscopic crystals.
Calcium oxalate crystals (choice B) are seen in primary oxalosis and are bipyramidal, positively birefringent crystals.
Calcium pyrophosphate dihydrate crystals (choice C) are a feature of pseudogout and are rod-to-rhomboidal-shaped, weakly positively birefringent crystals.
Cholesterol crystals (choice D) are seen in chronic and chylous effusions in inflammatory and degenerative arthritis, where they form large, flat, rhomboidal plates with notched corners.
--------------------------------------------------------------------------------
27>The correct answer is B. The innervation of the tongue is complex. The mandibular division of the trigeminal nerve (V3) carries general somatic sensation from the anterior two-thirds of the tongue.
The maxillary division (V2, choice A) carries somatic sensation from the palate, upper gums, and upper lip.
The facial nerve (VII, choice C) carries taste from the anterior two-thirds of the tongue.
The glossopharyngeal nerve (IX, choice D) carries sensation and taste from the posterior one-third of the tongue.
The vagus nerve (X, choice E) carries sensation from the lower pharynx.
--------------------------------------------------------------------------------
28>The correct answer is B. Brucella abortus produces a chronic, granulomatous disease with caseating granulomas. Most cases occur in four states (Texas, California, Virginia, and Florida), and are associated with cattle, in which it produces spontaneous septic abortions. Most cases of brucellosis produce mild disease or fevers of unknown origin. However, Brucella spp. can infect the cardiovascular system and cause a localized infection. B. abortus is the most common species to cause endocarditis. The aortic valve is most commonly involved, followed by the mitral valve, and then both valves. Most cases of brucellosis are associated with occupational exposure, in persons such as veterinarians, ranchers, and those who handle carcasses.
Bacillus anthracis (choice A) is the causative agent for anthrax. It usually produces cutaneous disease (malignant pustule or eschar) at the site of inoculation in handlers of animal skins. It can also produce a severe hemorrhagic pneumonia (Woolsorter's disease) and septicemia. At-risk groups include those who handle animal carcasses or skins.
Coccidioides immitis (choice C) is a dimorphic fungal disease producing a granulomatous pulmonary syndrome that is more severe in dark-skinned individuals. Disseminated disease occurs most often in Filipinos, Mexicans, and Africans. The infective form is the arthrospore; the diagnostic form in tissue is the spherule containing endospores. The disease is endemic in the San Joaquin River Valley. At-risk groups include military personnel, agricultural workers, construction workers, oil field workers, archaeology students, participants in outdoor sports, and sightseers. Remote infections from fomites (cotton harvested in the Southwestern U.S.) have been reported.
Erysipelothrix rhusiopathiae (choice D) is a pleomorphic, gram-negative rod that causes a localized skin infection. It is an occupational disease of fishermen, fish handlers, butchers, meat-processing workers, poultry workers, farmers, veterinarians, abattoir workers, and housewives.
Trichinella spiralis (choice E) is a nematode infection caused by the ingestion of larvae found in undercooked meat. Pork is the most common contaminated meat. However, outbreaks in the northern parts of the U.S. have been associated with eating undercooked infected bear meat. Symptoms include diarrhea, periorbital edema, myositis, fever, and eosinophilia.
--------------------------------------------------------------------------------
29>The correct answer is B. This patient has Tay-Sachs disease, an autosomal recessive disorder caused by the deficiency of hexosaminidase A, which leads to the accumulation of ganglioside GM2 in neurons, producing a degenerative neurologic disease. Children appear normal at birth, but then begin to suffer from diminished responsiveness, deafness, blindness, loss of neurologic function, and seizures. A cherry-red spot on the macula may be seen by ophthalmoscopic examination. Death usually occurs by 4 to 5 years of age. There is no therapy. The incidence is higher among Jews of Eastern European descent. Since the parents must be heterozygotes for the mutant hexosaminidase A allele, they would be expected to have diminished levels of the enzyme.
A defect in the dystrophin (choice A) gene produces Duchenne muscular dystrophy, characterized by onset of weakness in early childhood.
A severe deficiency in HGPRT (choice C) will lead to Lesch-Nyhan syndrome, characterized by excessive uric acid production, mental retardation, spasticity, self-mutilation, and aggressive, destructive behavior.
Deficiency of phenylalanine hydroxylase (choice D) results in classic phenylketonuria, a disease in which phenylalanine, phenylpyruvate, phenylacetate, and phenyllactate accumulate in plasma and urine. Clinically, there is a musty body odor and mental retardation.
Hypophosphatemic rickets is an X-linked dominant condition causing abnormal regulation of vitamin D3 (choice E) metabolism and defects in renal tubular phosphate transport. Symptoms include growth retardation, osteomalacia, and rickets.
--------------------------------------------------------------------------------
30>The correct answer is C. The fibrinolytic activity of streptokinase is due to its ability to bind and cleave plasminogen, producing plasmin. Plasmin directly cleaves fibrin, both between and within the fibrin polymers, thus breaking up thrombi and potentially restoring blood flow to ischemic cardiac muscle. This same mechanism of fibrinolysis is shared by urokinase and tissue-plasminogen activator (tPA).
Antithrombin III (choice A) is a coagulation inhibitor that binds to and inactivates thrombin. Antithrombin III is anticoagulant, not fibrinolyti

Reply
#12
1>A 37-year-old female presents to the emergency room with a fever.
Chest x-ray shows multiple patchy infiltrates in both lungs.
Echocardiography and blood cultures suggest a diagnosis of acute
bacterial endocarditis limited to the tricuspid valve. Which of
the following is the most probable etiology?
A. Congenital heart disease
B. Illicit drug use
C. Rheumatic fever
D. Rheumatoid arthritis
E. Systemic lupus erythematosus

Answer
--------------------------------------------------------------------------------

<2>Which of the following is the primary opsonin in the complement system?

A. C1q
B. C3b
C. C5
D. C5a
E. Factor B

Answer
--------------------------------------------------------------------------------

<3>A patient is referred to a neurologist because of ataxia. Neurological
examination reveals a loss of proprioception and a wide-based, slapping gate
Magnetic resonance imaging reveals degeneration of the dorsal columns and
dorsal roots of the spinal cord. Which of the following organisms is most
likely to have caused this pattern of damage?

A. Haemophilus influenzae
B. Herpes simplex I
C. Neisseria gonorrhoeae
D. Neisseria meningitidis
E. Treponema pallidum


Answer
--------------------------------------------------------------------------------

<4>A 24-year-old woman in her third trimester of pregnancy presents with
urinary frequency and burning for the past few days. She denies fever,
nausea, vomiting, or chills. She takes no medications besides prenatal
vitamins and is generally in good health. Physical examination is
remarkable for mild suprapubic tenderness, and a urine dipstick is
positive for white blood cells, protein, and a small amount of blood.
Culture produces greater than 100,000 colonies of gram-negative
bacilli. Which of the following attributes of this uropathogenic
organism is most strongly associated with its virulence?


A. Bundle-forming pili
B. GVVPQ fimbriae
C. Heat labile toxins
D. Heat stable toxins
E. P pili
F. Type 1 pili


Answer
--------------------------------------------------------------------------------

<5>A previously healthy 11-year-old girl develops a gastrointestinal
infection with cramping and watery stools. After several days, she begins
to pass blood per rectum, and is hospitalized for dehydration. In the
hospital, she is noted to have decreasing urine output with rising blood
urea nitrogen (BUN). Total blood count reveals anemia and
thrombocytopenia, and the peripheral smear is remarkable for
fragmented red cells (schistocytes). Infection with which of
the following bacterial genera is most likely responsible fo
r this syndrome?


A. Campylobacter
B. Clostridium
C. Salmonella
D. Shigella
E. Vibrio



Answer
--------------------------------------------------------------------------------

<6>Five days after returning to his military base in South Carolina after
survival training in the nearby countryside, an 18-year-old recruit reports
to the infirmary complaining of a headache. Physical examination reveals a
fever, but no other abnormalities are noted. A few days later he returns
to the infirmary with a maculopapular rash involving the hands and feet.
The rash then spreads centripetally to involve the trunk. Which of the
following diseases should be suspected?


A. Chickenpox
B. German measles
C. Measles
D. Mumps
E. Rocky Mountain spotted fever



Answer
--------------------------------------------------------------------------------

<7>A 32-year-old, blood type A positive male receives a kidney transplant
from a blood type B positive female donor with whom he had a 6-antigen HL
A match. Once the kidney is anastomosed to the man's vasculature,
the transplant team immediately begins to observe swelling and
interstitial hemorrhage. After the surgery, the patient developed
fever and leukocytosis and produced no urine. Which of the following
is the most likely explanation?
A. Acute rejection due to antibody-mediated immunity
B. Acute rejection due to cell-mediated immunity
C. Chronic rejection due to cell-mediated immunity to minor HLA antigens
D. Hyperacute rejection due to lymphocyte and macrophage infiltration
E. Hyperacute rejection due to preformed ABO blood group antibodies


Answer
--------------------------------------------------------------------------------

<8>A sexually active 18-year-old woman presents with a fever of 102 F for
the past 24 hours and lower abdominal pain and anorexia for the past 5 days.
On physical examination, there is generalized tenderness of the abdomen,
and the cervix is erythematous with motion tenderness. There is no rash
nor any lesions on the external genitalia. A smear of the odorless
cervical discharge contains sloughed epithelial cells and scant
neutrophils. Which of the following would likely be found in the
exudate?


A. A naked, icosahedral double-stranded circular DNA virus
B. Iodine-staining intraepithelial inclusion bodies
C. Intraneutrophilic gram-negative diplococci
D. Intranuclear "owl's eye" inclusion bodies
E. Lactose-fermenting gram-negative bacilli
F. Pear-shaped flagellated protozoa
G. Pleomorphic, gram-negative rods
H. Spirochetes on dark-field microscopy


Answer
--------------------------------------------------------------------------------

<9>A 45-year-old homeless man has a chronic cough, a cavitary lesion of the
lung, and is sputum positive for acid-fast bacilli. Which of the following
is the principle form of defense by which the patient's body fights this
infection?


A. Antibody-mediated phagocytosis
B. Cell-mediated immunity
C. IgA-mediated hypersensitivity
D. IgE-mediated hypersensitivity
E. Neutrophil ingestion of bacteria


Answer
--------------------------------------------------------------------------------

<10>A 15-year-old girl in a rural community has swollen, painful lymph nodes
in her right axilla. Physical examination reveals multiple scratches on her
right arm with a papule associated with one of the scratch marks. She
states that the scratches occurred about 5 days ago. What type of animal
is the most likely source of the infection?


A. Cat or kitten
B. Chicken
C. Dog or puppy
D. Horse
E. Parrot


Answer
--------------------------------------------------------------------------------

<11>A 38-year-old AIDS patient presents to the clinic complaining of nausea,
occasional vomiting and "bumps" on his groin. On physical examination,
multiple, nontender, pedunculated reddish purple nodules in the inguinal
and perirectal areas are observed. The patient's liver is palpable 8 cm
below the right costal margin. Routine laboratory tests are unremarkable
except for an alanine aminotransferase level of 58 and alkaline
phosphatase of 90. He denies any foreign travel, but has two pet cats
. Which of the following is the most likely cause of this patient's
infection?


A. Bartonella henselae
B. Human papillomavirus
C. Molluscum contagiosum virus
D. Rickettsia prowazekii
E. Treponema pallidum

Answer
--------------------------------------------------------------------------------

<12>A 49-year-old Vietnamese man is diagnosed with tuberculosis. On physical
examination, large flocculent masses are noted over the lateral lumbar back
, and a similar mass is located in the ipsilateral groin. This pattern of
involvement strongly suggests an abscess tracking along the
A. adductor longus
B. gluteus maximus
C. gluteus minimus
D. piriformis
E. psoas major

--------------------------------------------------------------------------------
Answer
<13>A 25-year-old man presents with a high fever and generalized malaise.
His condition deteriorates so rapidly that his friends decide to take him
to the emergency department 24 hours after the onset of symptoms. He has a
history of intravenous drug abuse. A test for anti-HIV antibodies is
negative. Physical examination reveals a systolic murmur, and
echocardiography shows bulky vegetations attached to the tricuspid valve
leaflets. Which of the following microorganisms will be most likely be
isolated from this patient's blood cultures?
A. Candida albicans
B. Hemophilus influenzae
C. Staphylococcus aureus
D. Staphylococcus epidermidis
E. Viridans (a-hemolytic) streptococci
Answer
--------------------------------------------------------------------------------

<14>A 23-year-old man develops explosive watery diarrhea with blood, fecal
leukocytes, and mucus approximately 3 days after eating chicken that was
improperly cooked. Comma-shaped organisms were found in the fecal smea
r along with red blood cells and leukocytes. Which of the following
pathogens is the most likely cause of these symptoms?
A. Campylobacter jejuni
B. Enterotoxigenic E. coli
C. Shigella sonnei
D. Staphylococcus aureus
E. Vibrio cholera

Answer
--------------------------------------------------------------------------------
<15>An otherwise healthy 3-year-old child is brought to the pediatrician
with umbilicated, flesh-colored papules on his trunk. This condition is
related to infection with which of the following viruses?
A. Cytomegalovirus
B. Herpesvirus 6
C. Parvovirus
D. Poxvirus
E. Variola
Answer
--------------------------------------------------------------------------------

<16>A 47-year-old male presents with declining renal function characterized
by oliguria, elevated blood urea nitrogen and creatinine, and hematuria .
He also complains of nasal congestion and epistaxis. Review of systems is
notable for occasional cough and hemoptysis. Examination shows mucosal
ulceration and nasal septal perforation, but no polyps. Which of the
following serum markers would likely be present in this case?
A. Anti-centromere antibody
B. Anti-Ro
C. Anti-SS-B
D. c-ANCA (cytoplasmic antinuclear cytoplasmic antibody)
E. Decreased erythrocyte sedimentation rate (ESR)
Answer
--------------------------------------------------------------------------------
<17>A child with sickle cell anemia is seen in a hematology clinic. Her
mother states that she has been feeling very tired lately, and may have
"come down with a virus." On physical examination, the girl is very pale,
and a complete blood count shows severe anemia. A bone marrow aspirate
contains no erythroid precursor cells. The girl was probably infected
with which of the following viruses?
A. Coxsackie virus
B. Echovirus
C. Hepadnavirus
D. Herpes virus
E. Parvovirus
Answer
--------------------------------------------------------------------------------
<18>Zygomycosis, a destructive fungal infection of the sinuses, is likely to
reach the brain by which of the following routes?
A. Cavernous sinus
B. External carotid artery
C. Internal carotid artery
D. Superior sagittal sinus
E. Superior vena cava
Answer
--------------------------------------------------------------------------------

<19>A newborn in the neonatal intensive care unit becomes tachypneic and
irritable. Blood cultures grow a gram-positive coccus in chains that is
presumed to be a streptococcus. Which of the following characteristics
would help to differentiate Streptococcus agalactiae from Streptococcus
pneumoniae?
A. Alpha-hemolysis
B. Carbohydrate capsule
C. Cytochrome enzyme system
D. Growth in bile
E. Oxacillin sensitivity
Answer
--------------------------------------------------------------------------------
<20>A 16-year-old girl presents with a painlessly enlarged lymph node in her
right axilla. Peripheral blood counts are within normal limits. The lymph
node is biopsied, and numerous granulomas filled with neutrophils and
necrotic debris are observed. Which of the following organisms could
produce this disease?
A. Bartonella henselae
B. Borrelia burgdorferi
C. Chlamydia psittaci
D. Coxiella burnetii
E. Rickettsia prowazekii
Answer
--------------------------------------------------------------------------------
<21>A 36-year-old mother of two children presents with a 4-day history of
swollen, painful hands. Her wrists and metacarpophalangeal joints are
boggy and inflamed bilaterally. Her 5-year-old son had been sent home
from school approximately 3 weeks previously with red cheeks and a
blotchy rash on his torso. What is the most likely diagnosis?
A. Listeriosis
B. Lyme disease
C. Mumps
D. Parvovirus
E. Reiter's syndrome
Answer
--------------------------------------------------------------------------------

<22>A 37-year-old, intravenous drug-abusing male presents with fever and
chills. Blood cultures are positive for Staphylococcus aureus. He develops
central nervous system symptoms, and a cerebral abscess is suspected.
Which part of the brain is most often affected by septic emboli in
patients with infective endocarditis?
A. Brainstem
B. Cerebellum
C. Frontal lobe
D. Occipital lobe
E. Parietal lobe
Answer
--------------------------------------------------------------------------------
<23>Which of the following organisms is the most common cause of
community-acquired pneumonia?
A. Chlamydia pneumoniae
B. Haemophilus influenzae
C. Mycoplasma pneumoniae
D. Staphylococcus aureus
E. Streptococcus pneumoniae
Answer
--------------------------------------------------------------------------------
<24>One week following a visit to the woods along an Eastern seaboard beach,
a 50-year-old woman develops fever, headache, chills, and fatigue. A blood
smear demonstrates protozoa within erythrocytes. Which of the following is
the most likely pathogen?
A. Babesia microti
B. Leishmania donovania
C. Plasmodium falciparum
D. Plasmodium vivax
E. Trypanosoma cruzi
Answer
--------------------------------------------------------------------------------
<25>A 20-year-old female presents with a two day history of dysuria and
increased urinary frequency. She states that she was recently married and
was not sexually active prior to the marriage. Physical exam reveals a
temperature of 100.7 °F with normal vital signs. Gynecological exam reveals
no evidence of discharge, vaginitis, or cervicitis. Urinalysis reveals 14
white blood cells per high-powered field with many gram-negative rods.
The most appropriate therapy would be
A. ampicillin
B. ceftriaxone
C. fluconazole
D. gentamicin
E. metronidazole
Answer
--------------------------------------------------------------------------------
<26>An active intravenous drug abuser presents to the emergency department
with fever of 5 days' duration, a cough occasionally productive of blood,
and pleuritic chest pain. Petechiae are present in his mouth and
conjunctivae, and splinter hemorrhages are visible under the fingernails.
Which of the following test results would most likely confirm the identity
of the causative agent?
A. Antibodies to p24 capsid antigen
B. Antibodies to Trichinella spiralis antigen
C. Blood culture of a catalase-positive, novobiocin-sensitive, gram-positive coccus
D. Blood culture of a coagulase-positive, catalase positive, gram-positive coccus
E. Blood culture of a gamma-hemolytic, gram-positive coccus on bile-esculin agar
F. Blood culture of an alpha-hemolytic, optochin-resistant, gram-positive coccus
G. Blood culture of an alpha-hemolytic, optochin-sensitive, gram-positive coccus
Answer
--------------------------------------------------------------------------------

<27>A 43-year-old executive presents to a physician with chronic, symmetric
polyarthritis involving the knees. The man gives a history of having
developed an extensive rash after a deer hunting trip in Connecticut
several years earlier. He recalls that he felt "sick" at the time,
and developed knee pain that prevented him from climbing stairs for
several months, but then partially resolved. Which of the following
organisms is most likely etiologically related to the patient's
arthritis?
A. Fungus
B. Gram-negative cocci
C. Gram-negative rod
D. Gram-positive cocci
E. Spirochete
Answer
--------------------------------------------------------------------------------
<28>A 34-year-old woman presents with fatigue, malaise, and swollen, tender
joints. Physical examination is significant for a maculopapular eruption
over sun-exposed areas, including the face. Examination of a peripheral
blood smear reveals mild thrombocytopenia. Which of the following
autoantibodies, if present, would be most specific for the diagnosis of
the patient's disorder?
A. Anti-centromere antibody
B. Anti-IgG antibody
C. Antinuclear antibody
D. Anti-Sm (Smith antigen) antibody
E. Anti-SS-A (Ro) antibody
Answer
--------------------------------------------------------------------------------
<29>A 33-year-old single mother of two young children visits her physician
because of an oral ulcer. A review of systems is significant for fatigue,
myalgia, and joint pain. Laboratory results demonstrate leukopenia, and
a high-titered antinuclear antibody. A speckled staining pattern due to
anti-Sm is seen with immunofluorescence; urinary protein is elevated.
Which of the following is the most likely diagnosis?
A. Generalized fatigue
B. Goodpasture's syndrome
C Mixed connective tissue disease
D. Scleroderma
E. Systemic lupus erythematosus
Answer
--------------------------------------------------------------------------------

<30>A 67-year-old black man with a history of glucose-6-phosphate
dehydrogenase (G-6-PD) deficiency presents with fever, irritative voiding
symptoms, and perineal pain. Rectal examination is remarkable for a boggy,
exquisitely tender prostate. A urine Gram's stain is positive for
gram-negative rods. The risk for development of hemolytic anemia is
highest if he receives high-dose, 21-day therapy with
A. ampicillin
B. cefaclor
C. ciprofloxacin
D. sulfamethoxazole/trimethoprim
E. tetracycline
Answer
--------------------------------------------------------------------------------
<31>A patient with colorectal cancer develops septicemia complicated by
endocarditis. You would expect the blood cultures to grow
A. Streptococcus agalactiae
B. Streptococcus bovis
C. Streptococcus pneumoniae
D. Streptococcus pyogenes
E. Streptococcus viridans
Answer
--------------------------------------------------------------------------------
<32>A patient with a cavitary lung lesion coughs up sputum that contains
thin, acid-fast positive rods. Which of the following features would most
likely be associated with these bacteria?
A. Nutritional requirement for factors V and X
B. Streptokinase
C. Toxic shock syndrome toxin
D. Visible under dark field illumination
E. Waxy envelope

Answer
--------------------------------------------------------------------------------
<33>A 27-year-old woman presents to the emergency department complaining of
10-12 episodes of nonbloody diarrhea per day for the past 2 days, along
with severe abdominal cramps, nausea, vomiting, and a low-grade fever.
She states that she just returned from a vacation to Mexico. While in
Mexico, she did not drink any of the local water and ate only cooked
foods and a few fresh salads. If fecal leukocytes are present, the
patient should most likely be empirically treated with
A. acyclovir
B. ciprofloxacin
C. mebendazole
D. quinine
E. tetracycline

Answer
--------------------------------------------------------------------------------

<34>A 54-year-old woman suffering from influenza deteriorates and develops
shaking chills and a high fever. Physical examination is remarkable for
dullness to percussion at the left base and decreased breath sounds on
the left. Chest x-ray confirms the diagnosis of lobar pneumonia, presumed
to be caused by Streptococcus pneumoniae. The patient has no known drug
allergies. Which of the following antibiotics would be most appropriate
to treat the patient's condition?
A. Cefotaxime
B. Chloramphenicol
C. Erythromycin
D. Penicillin
E. Vancomycin
Answer
--------------------------------------------------------------------------------

<35>A 4-year-old boy is seen by his pediatrician for epistaxis. The patient
has a history of multiple bacterial and viral respiratory tract infections
and eczema. An uncle had similar problems. Physical examination is
remarkable for multiple petechial lesions on the skin and mucous
membranes. Serum IgE is increased, and platelets are decreased.
Which of the following is the most likely diagnosis?
A. Acquired hypogammaglobulinemia
B. Ataxia telangiectasia
C. DiGeorge syndrome
D. Selective IgA deficiency
E. Wiskott-Aldrich syndrome
Answer
--------------------------------------------------------------------------------

<36>Which of the following organisms is most likely to be implicated as a
cause of urethritis that persists after antibiotic therapy for gonorrhea?
A. Actinomyces
B. Chlamydia
C. Mycobacteria
D. Nocardia
E. Rickettsia
Answer
--------------------------------------------------------------------------------
<37>A 73-year-old woman with a history of diabetes presents with left ear
pain and drainage of pus from the ear canal. She has swelling and tenderness
over the left mastoid bone. Which of the following microorganisms is the
most likely causative agent?
A. Hemophilus influenzae
B. Klebsiella pneumoniae
C. Mucor sp.
D. Pseudomonas aeruginosa
E. Streptococcus pyogenes
Answer
--------------------------------------------------------------------------------

<38>: A 25-year-old female presents with a confluent maculopapular rash that
began on her face, then spread downward over her trunk. She states that 3
days ago she started having a fever and headache, with bilateral pain
associated with the front and back of her neck. She also complains of
joint pain. Which of the following diseases does she most likely have?
A. Infectious mononucleosis
B. Lyme disease
C. Roseola
D. Rubella
E. Rubeola
Answer
--------------------------------------------------------------------------------


<39>A 24-year-old woman presents with a 3-day history of fever, chills, chest
pain, and cough productive of rust-colored sputum. Past medical history
includes a splenectomy 1 year ago. A chest x-ray film indicates
consolidation of the right lower lobe. Blood cultures are positive for
a-hemolytic gram-positive diplococci. Immunity to the causative organism
is based on
A. alternative complement pathway activation
B. antibody to an a-helical coiled fimbria
C. IgA antibodies to C carbohydrate
D. IgG antibodies to C carbohydrate
E. IgG antibodies to a surface acidic polysaccharide
Answer
--------------------------------------------------------------------------------
<40>A 3-year-old male presents with a skin rash and epistaxis. He has had
several, severe sinopulmonary infections. A careful history reveals that
his maternal grandfather died of bleeding complications following an
emergency cholecystectomy. What additional findings are likely in this
case?
A. A CD4/CD8 ratio of < 1.5:1
B. Cerebellar ataxia
C. Elevated platelet count and high serum IgG, IgA, and IgE levels
D. Low platelet count and low serum IgG levels
E. Low platelet count and low serum IgM levels
Answer
--------------------------------------------------------------------------------
<41> Which of the following characteristics most strongly suggests that
the cell depicted above is a phagocytically active macrophage?
A. Immunocytochemical detection of collagenase
B. Microvilli-covered surface
C. Presence of receptors for IgG and complement
D. Presence of secondary lysosomes throughout cytoplasm
E. Shape of the nucleus
Answer
--------------------------------------------------------------------------------

<42>A 27-year-old white male presents with a 3-week history of several
swollen and painful toes and knees. He has a past history of conjunctivitis. He also describes some low back stiffness that is more severe in the morning. Which of the following is the most likely diagnosis?
A. Gout
B. Lyme disease
C. Reiter's syndrome
D. Rheumatoid arthritis
E. Septic arthritis
Answer
--------------------------------------------------------------------------------
--------------------------------------------------------------------------------
<43>A 1-year-old girl presents with a 2-day history of fever,
vomiting, and watery, nonbloody diarrhea. On physical exam, she
appears dehydrated. Which of the following best describes the
most likely infecting organism?
A. It has a complex double-stranded DNA genome
B. It has a partially double-stranded circular DNA genome
C. It has a segmented, double-stranded RNA genome
D. It has a single-stranded circular RNA genome
E. It has a single-stranded RNA genome
Answer
--------------------------------------------------------------------------------
<44>A 46-year-old woman visits her podiatrist to have several bunions
removed from her right foot. She chooses conscious sedation rather
than general anesthesia for this procedure. She is given intravenous
midazolam to supplement the local anesthetics that are injected into
her foot. Midway through the surgery, she suddenly becomes agitated,
combative, and exhibits involuntary movements. The anesthesiologist
determines that she is having a paradoxical reaction to the midazolam
and immediately administers
A. flumazenil
B. glucagon
C. naloxone
D. nitrite
E. protamine
Answer
--------------------------------------------------------------------------------

<45>A 24-year-old woman attempts suicide by taking an overdose of diazepam.
She is rushed to the emergency department, where the attending physician
will most likely order which of the following treatments?
A. Acetylcysteine
B. Atropine
C. Bicarbonate
D. CaNa2EDTA chelation
E. Deferoxamine
F. Ethanol
G. Flumazenil
H. Physostigmine
I. Pralidoxime
J. Protamine
Answer
--------------------------------------------------------------------------------

<46>Which of the following enzymes is stimulated by glucagon?
A. Acetyl-CoA carboxylase
B. Glycogen phosphorylase
C. Glycogen synthase
D. HMG-CoA reductase
E. Pyruvate kinase
Answer
--------------------------------------------------------------------------------

<47>A 29-year-old epileptic sanitation engineer is maintained on primidone.
Ultrastructural examination of a liver biopsy reveals increased amounts of
smooth endoplasmic reticulum. This change is most closely related to
increases in the activity of which of the following?
A. P-450 system
B. Purine degradation
C. Pyrimidine synthesis
D. Tricarboxylic acid (Krebs) cycle
E. Urea cycle
Answer
--------------------------------------------------------------------------------

<48> A pregnant southeast Asian immigrant presents for prenatal
care. Her past medical history is significant for a severe illness 3
years ago characterized by fatigue, nausea, anorexia, vomiting, jaundice,
joint pains, and generalized skin lesions that slowly disappeared. She
has felt well recently. Which of the following laboratory tests should
be ordered to investigate the patient's past illness?

A. Hepatitis B surface antigen (HBsAg)
B. IgG cytomegalovirus (CMV) antibody levels
C. IgM antibody to HBsAg
D. IgM antibody to hepatitis B core antigen
E. Quantitation of hepatitis A virus (HAV) IgM antibody
Answer
--------------------------------------------------------------------------------

<49>A research scientist is studying calcium fluxes in cultured cells
using confocal laser scanning microscopy. The magnitude of the signal
(brightness) is proportional to the strength of the calcium flux.
Stimulation of which of the following receptor types would be expected
to produce the strongest signal?
A. Alpha-1 adrenergic receptor
B. Beta-1 adrenergic receptor
C. Dopamine-1 receptor
D. Muscarinic acetylcholine receptor
E. Nicotinic acetylcholine receptor

<50>A 10-year-old child is suspected of having pellagra because of chronic
symptoms including diarrhea, a red scaly rash, and mild cerebellar ataxia.
However, his diet is not deficient in protein and he appears to be
ingesting adequate amounts of niacin. A sister has a similar problem.
Chemical analysis of his urine demonstrates large amounts of free amino
acids. Which of the following is the most likely diagnosis?
A. Alkaptonuria
B. Carcinoid syndrome
C. Ehlers-Danlos syndrome
D. Hartnup's disease
E. Scurvy
Answer
--------------------------------------------------------------------------------



Answers
--------------------------------------------------------------------------------
--------------------------------------------------------------------------------
1>The correct answer is B. The most probable etiology of bacterial endocarditis involving the tricuspid valve is illicit intravenous drug use, which can introduce skin organisms into the venous system that then attack the tricuspid valve. Staphylococcus aureus accounts for between 60% and 90% of cases of endocarditis in intravenous drug users.
The endocarditis associated with congenital heart disease (choice A) typically involves either damaged valves or atrial or ventricular septal defects. The tricuspid valve is not particularly vulnerable.
Rheumatic fever (choice C) most commonly damages the mitral and aortic valves, and tricuspid damage is usually less severe and seen only when the mitral and aortic valves are heavily involved. Consequently, secondary bacterial endocarditis involving only the tricuspid valve in a patient with a history of rheumatic fever would be unusual.
Rheumatoid arthritis (choice D) is not associated with bacterial endocarditis.
Systemic lupus erythematosus (choice E) can produce small, aseptic vegetations on valves, but is not associated with bacterial endocarditis.
--------------------------------------------------------------------------------

2>The correct answer is B. C3b is the most critical molecule in both the classical and alternative complement pathways. C3, the most abundant protein of all the complement proteins, is cleaved into C3a and C3b. C3b attaches to bacterial surfaces for opsonization by phagocytes. C3a binds to mast cells and basophils, activating them and producing histamine release.
C1q (choice A) is a complement component in the classical pathway. It binds to the constant heavy domain of an IgG molecule that has reacted with the bacterial surface epitope. C1q is not involved in opsonization as it simply functions as an enzyme in the early complement cascade.
C5 (choice C) is a protein, that once split into C5b, initiates the assembly of the membrane attack complex. This complex consists of C5b, C6, C7, C8, and polymerization of C9, and is responsible for lysis of the bacteria. C5a is a strong chemotactic molecule.
This fragment is the result of C5 being split by the C5 convertases of both pathways. This C5a (choice D) is a strong chemotactic factor for neutrophils and results in stimulating the inflammatory response.
Factor B (choice E) is an activator protein of the alternative pathway. It combines with C3b to form C3bBb. This C3bBb is the C3 convertase of the alternative pathway.
--------------------------------------------------------------------------------

3>The correct answer is E. The findings described are those of tabes dorsalis, a form of tertiary syphilis caused by Treponema pallidum. Tabes dorsalis, and other forms of tertiary syphilis, are now uncommon in this country, possibly because the common use of antibiotics may "treat" many unsuspected cases of syphilis.
Haemophilus influenzae (choice A) and Neisseria meningitidis (choice D) can cause meningitis.
Neisseria gonorrhoeae (choice C) causes gonorrhea, which usually does not involve the CNS.
Herpes simplex I (choice B) can cause an encephalitis that typically involves the frontal and temporal lobes.
--------------------------------------------------------------------------------

4>The correct answer is E. Urinary tract infections are the most common bacterial infections encountered during pregnancy, and Escherichia coli is the most commonly isolated organism. 70% of cases in the U.S. are caused by P pili-positive strains.
Bundle-forming pili (choice A) are found in enteroaggregative E. coli (EAEC).
GVVPQ fimbriae (choice B) are found in EAEC.
Heat labile toxins (choice C) are pathogenic factors in enterotoxic strains (ETEC).
Heat stable toxins (choice D) are pathogenic factors in ETEC or EAEC.
Type 1 pili (choice F) are a major pathogenic factor in ETEC.
--------------------------------------------------------------------------------
5>The correct answer is D. This patient has developed hemolytic-uremic syndrome (HUS), a complication of the Shiga toxin or Shiga-like toxin: exotoxins released by Shigella species and the enterohemorrhagic E.coli. HUS in children usually develops after a gastrointestinal or flu-like illness, and is characterized by bleeding, oliguria, hematuria and microangiopathic hemolytic anemia. Presumably the Shiga toxin is toxic to the microvasculature, producing microthrombi that consume platelets and RBCs, and may fragment the red cell membrane.
The incorrect choices are all bacteria which may produce an enterocolitis, but do not elicit HUS.
A long-term consequence of Campylobacter (choice A) infection is a reactive arthritis or full-blown Reiter's syndrome.
Clostridial enterocolitis is produced by Clostridium difficile (choice B), a normal inhabitant of the gut that produces pseudomembranous colitis when other gut flora are suppressed by treatment with antibiotics.
In the United States, Salmonella infections (choice C) are almost all non-typhoid inflammatory diarrhea, producing a simple enterocolitis that may proceed to sepsis in some cases. Typhoid fever (produced by Salmonella typhi and S. paratyphi) produces a protracted illness that progresses over several weeks and includes rash and very high fevers, but not HUS.
Vibrio (choice E) infections produce copious amounts of watery diarrhea, and the major risk of cholera and other Vibrio enteritides is shock due to hypovolemia or electrolyte loss.
--------------------------------------------------------------------------------
6>The correct answer is E. Take rashes involving the palms and soles (otherwise unusual sites) very seriously: only a small number of infections can cause this pattern, including Rocky Mountain spotted fever, meningococcemia, and secondary syphilis. Rocky Mountain spotted fever is caused by the rickettsia R. rickettsii, and is found throughout the United States, particularly in south central and eastern portions (not the Rocky Mountains). 3-12 days after a tick bite, patients develop malaise, frontal headache, and fever. Several days later, the rash described in the question stem develops. Other manifestations can include hepatosplenomegaly, thrombocytopenia, and (potentially fatal) disseminated intravascular coagulation.
Chickenpox (choice A), or varicella, is characterized by maculopapules that evolve into vesicles over hours to days, then eventually form crusts. Typical lesions first appear on the trunk and face and rapidly spread to involve other areas.
The maculopapular rash of German measles (choice B), or rubella, usually begins on the face, then spreads down the body.
Although the maculopapular rash of measles (choice C), or rubeola, can include the palms and soles, it typically begins along the hairline in frontal and temporal regions, then spreads down the trunk to the limbs.
Mumps (choice D) is characterized by fever, malaise, parotitis, and orchitis, but not a rash.
--------------------------------------------------------------------------------
7>The correct answer is E. The patient is suffering from hyperacute rejection due to the preformed anti-B ABO blood group antibody found in all type A positive individuals. Hyperacute rejection occurs within minutes to a few hours of the time of transplantation, and is due to the destruction of the transplanted tissue by preformed antibodies reacting with antigens found on the transplanted tissue that activate complement and destroy the target tissue. Preformed antibodies can also be due to presensitization to a previous graft, blood transfusion, or pregnancy.
Acute rejection due to antibody-mediated immunity (choice A) is incorrect because this patient suffered from hyperacute rejection (immediate) occurring within minutes to hours, rather than days.
Acute rejection due to cell-mediated immunity (choice B) will not occur until several days or a week following transplantation. Acute rejection is due to type II and type IV reactions.
Chronic rejection, due to the presence of cell-mediated immunity to minor HLA antigens (choice C), occurs in allograft transplantation months to even years after the transplant. Chronic rejection is generally caused by both humoral and cell-mediated immunity.
An accelerated acute rejection, occurring in 3-5 days, can be caused by tissue infiltration and destruction by presensitized T lymphocytes and macrophages (choice D) and/or antibody-dependent, cell-mediated cytotoxicity (ADCC). Note that this is not a hyperacute reaction.
--------------------------------------------------------------------------------
8>The correct answer is B. The presentation is typical for pelvic inflammatory disease (PID). Chlamydia trachomata (serotypes D-K) is the most common bacterial cause of sexually transmitted disease (STD) in this country and is the most likely agent on the list to produce the symptoms described. It is an ATP-defective organism that must therefore live intracellularly in the human host and can be visualized inside epithelial cells with iodine, Giemsa, or fluorescent-antibody stains. The remainder of the answer choices refer to other agents that could be found in the female genital tract, either by sexual transmission or by contamination with fecal flora, but they are not the best choices.
A naked, icosahedral double-stranded circular DNA virus (choice A) refers to human papilloma virus, which is the most common cause of STDs in the U.S., but presents with anogenital warts.
Intraneutrophilic gram-negative diplococci (choice C) refers to Neisseria gonorrhoeae, which would be expected to present with dysuria and neutrophilic exudate.
Intranuclear "owl's eye" inclusion bodies (choice D) refers to cytomegalovirus, a common STD in the United States, but not a common agent of PID. Most cases in average adults are manifested by mononucleosis-like symptoms.
Lactose-fermenting gram-negative bacilli (choice E) would be consistent with Escherichia coli. Although this organism is the most common cause of urinary tract infections in women in the United States, it would not be expected to cause PID.
Pear-shaped flagellated protozoa (choice F) refers to the protozoan parasite Trichomonas vaginalis, the only protozoan STD in the world. Infection would be characterized by a malodorous, cheesy exudate, and there would be more erythema of the external genitalia than of the cervix.
Pleomorphic, gram-negative rods (choice G) are consistent with Hemophilus ducreyi, which causes chancroid, and presents in a different manner.
Spirochetes on dark-field microscopy (choice H) refers to Treponema pallidum, the causative agent of syphilis, which would produce rash and/or chancre, depending on the stage of the infection.
--------------------------------------------------------------------------------
9>The correct answer is B. The principle host defense in mycobacterial infections (such as this patient's tuberculosis) is cell-mediated immunity, which causes formation of granulomas. Unfortunately, in tuberculosis and in many other infectious diseases characterized by granuloma formation, the organisms may persist intracellularly for years in the granulomas, only to be a source of activation of the infection up to decades later.
While antibody-mediated phagocytosis (choice A) is a major host defense against many bacteria, it is not the principle defense against Mycobacteria.
IgA-mediated hypersensitivity (choice C) is not involved in the body's defense against Mycobacteria.
IgE-mediated hypersensitivity (choice D) is not involved in the body's defense against Mycobacteria. It is important in allergic reactions.
Neutrophil ingestion of bacteria (choice E) is a major host defense against bacteria, but is not the principle defense against Mycobacteria.
--------------------------------------------------------------------------------
10>The correct answer is A. This patient has the classic symptoms of cat scratch disease caused by the bacillus Bartonella henselae. The disease is self-limited with the onset of symptoms occurring 3-10 days following an inoculating scratch. The organism can be isolated from kittens, typically less than 1 year of age, or from fleas. A history of a new kitten in the house and the papule at the site of a scratch with regional painful adenopathy defines the classic scenario.
Chickens (choice B) can harbor Salmonella spp. producing a gastroenteritis or enterocolitis. Chicken guano is also a favorable environment for the fungus Histoplasma capsulatum. The mycelial phase thrives in the rich soil. The human disease is a granulomatous infection involving the lungs and mimicking tuberculosis.
Dogs or puppies (choice C) carry Capnocytophaga canimorsus as part of the normal flora of the oral cavity. Infections from licking or biting range from a self-limited cellulitis to fatal septicemia. Patients at risk for more severe infections are those with asplenia, alcoholism, or hematologic malignancies. This organism is also associated with cat bites, but the patient develops cellulitis and fulminant septicemia, especially in asplenic patients. Pasteurella multocida is another pathogen that colonizes the nasopharynx and gastrointestinal tract of cats and dogs. Cats have the highest rate of colonization (50-90%), followed by dogs (50%), swine (50%), and rats (14%). P. multocida most commonly causes a localized soft tissue infection or cellulitis after an animal bite, but systemic symptoms may be present in about 40% of the cases. These symptoms include osteomyelitis, septic arthritis, or tenosynovitis.
Horses (choice D) and horse manure have been associated with a pulmonary opportunistic infection with cavitation caused by Rhodococcus equi that resembles tuberculosis in immunocompromised patients. Burkholderia mallei (the cause of glanders) is characterized by non-caseating granulomatous abscesses of skin, lymphadenopathy, and pronounced involvement of the lungs.
Parrots (choice E) are associated with psittacosis caused by Chlamydia psittaci. Psittacosis is associated with a dry, hacking cough productive of scant sputum, an interstitial infiltrate in the lungs, severe headache, and myalgias. A pale macular rash is also seen.
--------------------------------------------------------------------------------
11>The correct answer is A. Bacillary angiomatosis is a disease that occurs primarily in AIDS patients, and is indicative of a defect in cell-mediated immunity. It is caused by either Bartonella henselae or Bartonella quintana. The domestic cat is the reservoir for these organisms and they are usually transmitted to humans via a cat scratch or cat bite. Patients with this illness usually have multiple skin lesions and extracutaneous manifestations involving liver and bone. Diagnosis is usually based on characteristic histopathologic findings including plump "epithelioid" endothelial cells and mitotic figures. A macrolide, such as erythromycin or azithromycin, is the drug of choice for the infection.
Human papillomavirus (choice B) causes warts. Infection can present as a sessile wart or as condyloma acuminatum, which are fleshy soft growths that coalesce into large masses. When cellular immunity is depressed, as in AIDS, the condylomata acuminatum proliferate.
Molluscum contagiosum virus (choice C) is a pox virus that is spread by close person-to-person contact. Infection produces a firm nodule that often becomes umbilicated, and may resolve by discharging its contents. In AIDS, the lesions do not resolve, but enlarge and spread.
Rickettsia prowazekii (choice D) is the cause of epidemic typhus. It is spread by the human body louse, Pediculus humanis. Its reservoirs are humans and flying squirrels.
Treponema pallidum (choice E) is the spirochete that causes syphilis. The characteristic primary lesion is a chancre (a painless, indurated ulcer) at the site of inoculation.
--------------------------------------------------------------------------------
12>The correct answer is E. This is the classic presentation of a psoas abscess. This clinical entity was formerly a fairly common complication of vertebral tuberculosis, but is now rare in clinical practice in this country. The psoas muscle is covered by a fibrous sheath known as the psoas fascia. This sheath is open superiorly, permitting an infection involving the soft tissues around the spine to enter the sheath, then track down to the groin.
The adductor longus (choice A) is a muscle of the anterior thigh, and is not related to the lumbar portion of the back.
The gluteus maximus (choice B) gluteus minimus (choice C) and piriformis (choice D) are muscles of the buttock with no relationship to the groin.
--------------------------------------------------------------------------------
13>The correct answer is C. The patient has a fever and is extremely ill. The most important clue to the diagnosis is the presence of bulky vegetations on the tricuspid valve, indicating that he has infective endocarditis. On the basis of the rapid clinical course, this is likely a case of acute infective endocarditis. The diagnosis of this condition must be confirmed by blood cultures, which are also necessary to determine bacterial antibiotic sensitivity. S. aureus (commonly present on the skin) is the most frequent etiologic agent of infective endocarditis in intravenous drug abusers. It commonly affects the tricuspid valve. Because of its high virulence, S. aureus-related endocarditis follows an acute course and may lead to death within a few days.
The causative agents of infective endocarditis differ depending on host factors. Fungal organisms, such as Candida albicans (choice A), may cause infective endocarditis in severely immunosuppressed patients, such as those with AIDS.
A minority of cases of infective endocarditis are caused by a number of normal commensals in the oral cavity, i.e., the "HACEK" group: Hemophilus (choice B), Actinobacillus, Cardiobacterium, Eikenella, and Kingella.
S. epidermidis (choice D) and other coagulase-negative staphylococci tend to produce endocarditis in recipients of prosthetic valves.
Viridans streptococci (choice E) are the most frequent agents causing endocarditis in previously abnormal valves, such as those damaged by rheumatic disease, or congenitally abnormal valves. Coagulase-negative staphylococci and viridans (a-hemolytic) streptococci are less virulent than S. aureus and are thus associated with a subacute (more prolonged) clinical course and a better prognosis
--------------------------------------------------------------------------------
14>The correct answer is A. Campylobacter jejuni is a pathogen causing an invasive enteric infection associated with ingestion of raw or undercooked food products, or through direct contact with infected animals. In the U.S., ingestion of contaminated poultry that has not been sufficiently cooked is the most common means of acquiring the infection. The patients typically have bloody diarrhea, abdominal pain, and fever. The presence of fecal leukocytes indicates an invasive infection. The organism is a gram negative rod with a "comma-shape."
Enterotoxigenic E. coli (choice B) causes the classic traveler's diarrhea. The infection is non-invasive and is acquired via the fecal-oral route through consumption of unbottled water or uncooked vegetables. The major manifestation is a copious outpouring of fluid from the GI tract presenting as explosive diarrhea. This is due to the action of one of two types of enterotoxins on the GI tract mucosa.
Shigella sonnei (choice C) produces a syndrome very similar to C. jejuni. However, the organism appears as a gram-negative rod on Gram's stain. It does not have a comma shape. Transmission is from person to person via the fecal-oral route. Infection requires a low infective dose since the organism is fairly resistant to gastric acidity.
Staphylococcus aureus (choice D) produces food poisoning due to the ingestion of a pre-formed enterotoxin. The organism is present in food that is high in salt content such as potato salad, custard, milk shakes, and mayonnaise. The patient presents with nausea, vomiting, and abdominal pain, followed by diarrhea beginning 1-6 hours after ingestion of the enterotoxin.
Vibrio cholerae (choice E) produces a secretory diarrhea due to increases in cAMP in the intestinal cells. The organism is not invasive. The patient presents with the sudden onset of painless, watery diarrhea that becomes voluminous, followed by vomiting. The stool appears nonbilious, gray, and slightly cloudy with flecks of mucus, no blood, and a sweet odor.
--------------------------------------------------------------------------------
15>The correct answer is D. The lesions are characteristic of molluscum contagiosum, which is a typically benign and self-limited condition caused by a poxvirus. The disease can be transmitted either venereally or through non-venereal contact. The other viruses listed do not cause similar skin lesions. Patients with advanced HIV infection may develop a severe, generalized, and persistent eruption, often involving the face and upper body.
Cytomegalovirus (choice A) causes congenital infections and disseminated infections in immunosuppressed patients.
Herpesvirus 6 (choice B)causes roseola (exanthem subitum).
Parvovirus (choice C) causes aplastic crises in patients with hemolytic anemia.
Variola (choice E) is the smallpox virus.
--------------------------------------------------------------------------------

16>The correct answer is D. This patient has Wegener's granulomatosis, which is characterized by renal involvement, severe upper respiratory tract symptoms, and pulmonary involvement. Other organ systems may also be involved. The renal syndrome is a crescentic rapidly progressive glomerulonephritis leading to renal failure. The upper respiratory tract findings include sinus pain and drainage, and purulent or bloody nasal discharge with or without nasal ulcerations. Nasal septal perforation may follow. Pulmonary involvement may be clinically silent with only infiltrates present on x-ray, or it may present as cough and hemoptysis. c-ANCA is a marker for Wegener's granulomatosis, present in a high percentage of patients.
Anti-centromere antibody (choice A) is associated with approximately 90% of cases with CREST syndrome (calcinosis, Raynaud's phenomenon, esophageal motility syndrome, sclerodactyly, and telangiectasia) which is also called limited scleroderma.
Anti-Ro (choice B) is also called anti-SS-A and is associated with Sjِgren's syndrome (70-95%).
Anti-SS-B (choice C) is associated with Sjِgren's syndrome (60-90%).
Decreased ESR (choice E) is not a marker of Wegener's. Instead, a markedly elevated ESR is seen. Additionally, mild anemia, thrombocytosis, leukocytosis, mild hypergammaglobulinemia (IgA), and mildly elevated rheumatoid factor are seen in this disorder.
--------------------------------------------------------------------------------
17>The correct answer is E. Parvoviruses are small single-stranded DNA viruses, of which only serotype B19 is pathogenic for humans. This virus causes three distinct syndromes: a childhood febrile rash known as erythema infectiosum ("Fifth disease"); aplastic crisis in individuals with chronic hemolytic diseases (sickle cell anemia, thalassemia, etc); and congenital infections that can present as stillbirth, hydrops fetalis (analogous to severe Rh incompatibility), or severe anemia.
Coxsackie viruses (choice A) usually cause cold-like illness, but can cause herpangina, myocarditis, and meningitis.
Echoviruses (choice B) can infect a variety of organ systems (GI, CNS, eyes, heart, respiratory, skin), but are not a cause of aplastic crises.
Hepadnavirus (choice C) is the causative agent of hepatitis B.
Herpes viruses (choice D) cause a variety of acute to chronic infections including herpes simplex types I and II, chicken pox, chronic herpes zoster, CMV infection, and Epstein Barr virus infections.
--------------------------------------------------------------------------------
18>The correct answer is A. The cavernous sinuses are located on either side of the body of the sphenoid bone, and become a potential route of infection because they receive blood both from the face (via the ophthalmic veins and sphenoparietal sinus) and some of the cerebral veins. The spread of infection, especially by Mucor sp., into the cavernous sinus, can produce either CNS infection or cavernous sinus thrombosis, both of which are potentially fatal.
The route from the face to the brain is not arterial (choices B and C).
The superior sagittal sinus (choice D) is located in the falx cerebri, and drains venous blood from the brain to other dural sinuses, from which it eventually drains into the jugular vein. Zygomycosis does not reach the brain by way of the superior sagittal sinus.
The superior vena cava (choice E) drains blood from the upper part of the body into the heart.
--------------------------------------------------------------------------------
19>The correct answer is A. Streptococci are usually initially speciated by their hemolytic capacity on sheep blood agar. Beta-hemolytic streptococci include groups A, B, and D. S. agalactiae is the classic group B streptococcus. The non beta-hemolytic streptococci consist principally of the pneumococci and the viridans group.
Both S. agalactiae and pneumococcus have a carbohydrate capsule (choice B), an important virulence factor and means of subtyping streptococcal species.
None of the streptococci utilize cytochrome enzymes (choice C). They derive all of their energy from the fermentation of sugars to lactic acid.
Neither pneumococcus nor S. agalactiae can grow in bile (choice D). This ability is specific for the enterococcus group (group D) of streptococci.
Both pneumococci and S. agalactiae are usually treated with penicillin-type antibiotics, although group B streptococci require a penicillinase-resistant type such as oxacillin (choice E).
--------------------------------------------------------------------------------
20>The correct answer is A. Bartonella henselae is the infective agent of cat scratch disease, which generally presents as regional lymphadenopathy with or without low fevers and headaches. Bartonella is a gram-variable pleomorphic rickettsial organism that is introduced to the skin in a cat bite or scratch. It produces a self-limited granulomatous response in the draining lymph nodes.
Borrelia burgdorferi (choice B) is a spirochetal organism that is transmitted by a tick bite (Ixodes spp.), producing Lyme disease. Lyme disease progresses from a skin rash to fevers, headache and pain over about one month. It may produce lymphadenopathy, but is not associated with granuloma formation.
Chlamydia psittaci (choice C) infection occurs after contact with infected bird droppings and produces an atypical pneumonia. The central nervous system may also be involved, but lymph nodes are spared. Chlamydia trachomatis is the chlamydial species that typically produces suppurative nodal granulomas (lymphogranuloma venereum).
Coxiella burnetii (choice D) infection is transmitted by inhaling dusts or drinking milk from infected mammals, especially sheep and cows. The disease in humans, Q fever, is marked by mild nonspecific symptoms or pneumonia, and may progress to myocarditis or hepatitis.
Rickettsia prowazekii (choice E) produces epidemic (louse-borne) typhus, which is transmitted by body lice and produces a rash akin to Rocky Mountain spotted fever. Although the organism may reside in the lymph nodes in dormancy, it does not elicit granuloma formation.
--------------------------------------------------------------------------------
21>The correct answer is D. Parvovirus B19 causes erythema infectiosum, or Fifth disease. The 5-year-old boy has the classic "slapped cheek" appearance. Adults typically do not get the facial rash, but have arthralgias and arthritis. The symmetrical distribution of involved joints is similar to that in rheumatoid arthritis. The onset in adults is typically 3 to 4 weeks after exposure. Parvovirus infections may persist in immunosuppressed patients, resulting in red blood cell aplasia.
Listeriosis (choice A) is caused by the gram-positive rod Listeria monocytogenes. Meningitis and bacteremia are common clinical manifestations. Elderly, neonates, pregnant women, and those taking steroids have the highest risk for infection.
Lyme disease (choice B) is caused by Borrelia burgdorferi. It is the most common vector-borne disease (Ixodes ticks) in the U.S. The incidence is highest in the summer and fall. The white-footed mouse and the white-tailed deer are zoonotic reservoirs. From 3 to 32 days following a tick bite, the patient develops fever, lymphadenopathy, meningismus, and the characteristic rash (erythema migrans). The rash enlarges and resolves over 3 to 4 weeks. Sequelae include arthritis, carditis, and neurologic abnormalities.
Mumps (choice C) is caused by a Paramyxovirus. The virus most commonly affects glandular tissue. Parotitis, pancreatitis, and orchitis are characteristic. Mumps meningoencephalitis is one of the most common viral meningitides. Mumps polyarthritis is most common in men between the ages of 20 and 30 years. Joint symptoms begin 1 to 2 weeks after the parotitis subsides and large joints are involved.
Reiter's syndrome (choice E) is a seronegative, asymmetric arthropathy predominantly affecting the lower extremities. It may be triggered by a C. trachomatis infection. In addition to the arthritis, patients may have urethritis (which is usually due to chlamydia), conjunctivitis, mucocutaneous disease such as balanitis, oral ulcerations, or keratoderma. Approximately 80% of patients are HLA-B27 po
--------------------------------------------------------------------------------
22>The correct answer is E. Embolization from infective endocarditis typically causes multiple, small parietal lobe abscesses. This "factoid" is worth knowing because some patients with infective endocarditis present with what clinically looks like multiple small "strokes", and their treatable cardiac disease may be completely unsuspected.
--------------------------------------------------------------------------------
23>The correct answer is E. The most common bacteria implicated in community-acquired pneumonia is the pneumococcus, Streptococcus pneumoniae. Other organisms frequently implicated in patients less than age 60 without comorbidity include Mycoplasma pneumoniae, respiratory viruses, Chlamydia pneumoniae, and Haemophilus influenzae. When community-acquired pneumonia occurs in elderly patients or patients with comorbidity, aerobic gram-negative bacilli and Staphylococcus aureus are added to the list.
The organisms listed in choices A, B, and C are important causes of community-acquired pneumonia, but are not the most frequent causes.
Staphylococcus aureus (choice D) is an important cause of community-acquired pneumonia (particularly in the elderly and in patients with comorbidity), but is not the most frequent cause.
--------------------------------------------------------------------------------
24>The correct answer is A. The combination of a one week incubation period, the Eastern seaboard clue, and the intra-erythrocyte parasites strongly suggest Babesia microti as the infecting organism. The clinical disease is called babesiosis. The infecting protozoan is related to Plasmodium and is transmitted by the bite of the Ixodes dammini tick. Occasional cases have been transmitted by blood transfusions. Most patients are asymptomatic; symptomatic cases in reasonably healthy individuals have the features listed in the question stem. Rare severe cases, which may be fatal, may develop in severely debilitated or asplenic individuals and can be accompanied by severe hemolysis (up to 30% of RBCs may have the parasites) with subsequent hemoglobinuria, hemolysis, and renal failure.
Leishmania donovania (choice B) is an intracellular tissue protozoan, not a blood protozoan.
Plasmodium falciparum (choice C) and Plasmodium vivax (choice D) are malarial parasites not encountered on the Eastern seaboard.
Trypanosoma cruzi (choice E) is an intracellular tissue protozoan, and does not infect blood cells.
--------------------------------------------------------------------------------
25>The correct answer is A. The patient's presentation is consistent with a simple urinary tract infection; there is a short history of dysuria, increased urinary frequency and the appearance of white blood cells and gram-negative rods in the urine. Urinary tract infections are common in women after they become sexually active. The infection is likely caused by urethral trauma during intercourse, which leads to bacterial contamination of the bladder. Since the majority of these infections are caused by Escherichia coli (a gram-negative rod), the most appropriate therapy would be ampicillin for around 10 days.
Ceftriaxone (choice B) is the treatment of choice for uncomplicated infections with N. gonorrhoeae, now that most strains are resistant to penicillin. Intravenous ceftriaxone is a regimen reserved for the treatment of life-threatening infections.
Fluconazole (choice C) is indicated for the treatment of vaginal candidiasis. Since there is no vaginal discharge and the patient has gram-negative rods in the urine, a diagnosis of vaginal candidiasis can be excluded.
Gentamicin (choice D) would be an i


Reply
#13
what is the source of this questions???????
Reply
« Next Oldest | Next Newest »


Forum Jump: